SlideShare a Scribd company logo
1 of 45
Krok 2 – 2014 Gynecology Base
1. A 52-year-old woman suffering from obesity, complains of bloody discharges
from sexual paths during 4 days. Last normal menses were 2 years ago.
Histological investigation of biopsy of the endometrium has revealed
adenomatous hyperplasia. What reason from the mentioned below caused the
development of disease?
A. Excessive transformation of preandrogens from adipose tissues
B. The increased contents of follicle-stimulating hormone
C. Supersecretion of androgens by the cortex of paranephroses.
D. Hypersecretion of estrogens by tissues of the organism.
E. Poor aromatization of preandrogens due to hypothyroidism
2. A 40-year-old woman complains of colic pains in the lower part of abdomen
and abundant bloody discharges from genital tract. Last 2 years she had
menses for 15-16 days, abundant, with clots, painful. Had 2 medical abortions.
In bimanual investigation: from the canal of the cervix uteri - a fibromatous
node, 3 cm in diameter, on the thin stem. Discharges are bloody,
moderate.Choose the correct tactics.
A. Operation: untwisting of born node
B. Phase by phase vitamin therapy
C. Hysterectomy without ovaries
D. Supravaginal ablation of the uterus without ovaries
E. Hormonal hemostasis
3. A 40-year-old woman complains of yellow color discharges from the vagina.
Bimanual examination: no pathological changes. Smear test: Trichomonas
vaginalis and mixed flora. Colposcopy: two hazy fields on the front labium,
with a negative Iodum probing. What is your tactics?
A. Treatment of specific colpitis with the subsequent biopsy
B. Cervix ectomy
C. Cryolysis of cervix uteri
D. Diathermocoagulation of the cervix uteri
E. Specific treatment of Trichomonas colpitis
4. A 32 y.o. woman consulted a gynecologist about having abundant long menses
within 3 months. Bimanual investigation: the body of the uterus is enlarged
according to about 12 weeks of pregnancy, distorted, tuberous, of dense
consistence. Appendages are not palpated. Histological test of the uterus body
mucosa: adenocystous hyperplasia of endometrium. Optimal medical tactics:
A. Surgical treatment
B. Radial therapy
C. Phase by phase vitamin therapy
D. Hormonetherapy
E. Phytotherapy
5. A woman complains of having slight dark bloody discharges and mild pains in
the lower part of abdomen for several days. Last menses were 7 weeks ago.
The pregnancy test is positive. Bimanual investigation: the body of the uterus
indicates for about 5-6 weeks of pregnancy, it is soft, painless. In the left
appendage there is a retort-like formation, 7х5 cm large, mobile, painless.
What examination is necessary for detection of fetus localization?
A. Ultrasound
B. Colposcopy
C. Cystoscopy
D. Hysteroscopy
E. Hromohydrotubation
6. A woman was hospitalised with full-term pregnancy. Examination: the uterus
is tender, the abdomen is tense, cardiac tones of the fetus are not auscultated.
What is the most probable complication of pregnancy?
A. Premature detachment of normally posed placenta
B. Premature labor
C. Acute hypoxia of a fetus
D. Hydramnion
E. Back occipital presentation
7. By the end of the 1st period of physiological labor clear amniotic fluid came
off. Contractions lasted 35-40 sec every 4-5min. Heartbeat of the fetus was
100 bpm. The BP was 140/90 mm Hg. What is the most probable diagnosis?
A. Acute hypoxia of the fetus
B. Premature labor
C. Back occipital presentation
D. Hydramnion
E. Premature detachment of normally posed placenta
8. A pregnant woman in her 40th week of pregnancy undergoes obstetric
examination: the cervix of uterus is undeveloped. The oxytocin test is
negative. Examination at 32 weeks revealed: AP 140/90 mm Hg, proteinuria 1
g/l, peripheral edemata. Reflexes are normal. Choose the most correct tactics:
A. Labour stimulation after preparation
B. Complex therapy of gestosis for 2 days
C. Absolute bed rest for 1 month
D. Caesarian section immediately
E. Complex therapy of gestosis for 7 days
9. Which gestational age gives the most accurate estimation of weeks of
pregnancy by uterine size?
A. Less that 12 weeks
B. Between 21 and 30 weeks
C. Between 12 and 20 weeks
D. Between 31 and 40 weeks
E. Over 40 weeks
10.A 26 year old woman had the second labour within the last 2 years with
oxytocin application. The childs weight is 4080 g. After the placent birth there
were massive bleeding, signs of hemorrhagic shock. Despite the injection of
contractive agents, good contraction of the uterus and absence of any cervical
and vaginal disorders, the bleeding proceeds. Choose the most probable cause
of bleeding:
A. Atony of the uterus
B. Hypotonia of the uterus
C. Hysterorrhexis
D. Injury of cervix of the uterus
E. Delay of the part of placenta
11. A woman is admitted to maternity home with discontinued labor activity and
slight bloody discharges from vagina. The condition is severe, the skin is pale,
consciousness is confused. BP is 80/40 mm Hg. Heartbeat of the fetus is not
heard. There was a Cesarian section a year ago. Could you please determine
the diagnosis?
A. Hysterorrhesis
B. Cord presentation
C. Expulsion of the mucous plug from cervix uteri
D. Premature expulsion of amniotic fluid
E. Placental presentation
12. On the first day after labour a woman had the rise of temperature up to 39°C.
Rupture of fetal membranes took place 36 hours before labour. Examination
of the bacterial flora of cervix of the uterus revealed hemocatheretic
streptococcus of A group. The uterus body is soft, tender. Discharges are
bloody, with admixtures of pus. Specify the most probable postnatal
complication:
A. Metroendometritis
B. Infective contamination of the urinary system
C. Apostasis of sutures after the episiotomy
D. Thrombophlebitis of veins of the pelvis
E. Infectious hematoma
13. Rise in temperature up to 39°C was registered the next day after a woman had
labor. Fetal membranes rupture took place 36 hours prior to labors. The
examination of the bacterial flora of cervix uterirevealed the following:
haemolytic streptococcus of group A. The uterus tissue is soft, tender.
Discharges are bloody, with mixing of pus. Establish the most probable
postnatal complication.
A. Metroendometritis
B. Infective contamination of the urinary system
C. Apostatis of stitches after the episiotomy
D. Thrombophlebitis of veins of the pelvis
E. Infected hematoma
14. A woman of a high-risk group (chronic pyelonephritis in anamnesis) had
vaginal delivery. The day after labour she complained of fever and loin pains,
frequent urodynia. Specify the most probable complication:
A. Infectious contamination of the urinary system
B. Infectious hematoma
C. Apostasis of sutures after episiotomy
D. Endometritis
E. Thrombophlebitis of veins of the pelvis
15. 13 months after the first labor a 24-year-old patient complained of
amenorrhea. Pregnancy ended in Caesarian section because of premature
detachment of normally positioned placenta which resulted in blood loss at
the rate of 2000 ml owing to disturbance of blood clotting. Choose the most
suitable investigation:
A. Estimation of gonadotropin rate
B. Progesteron assay
C. USI of small pelvis organs
D. Computer tomography of head
E. Estimation of testosteron rate in blood serum
16. In 13 months after the first labor a 24-year-old woman complains of
amenorrhea. Cesarian section was conducted as a result of premature
detachment of normally posed placenta. Hemorrhage has made low fidelity of
2000 ml due to breakdown of coagulation of blood. Choose the most suitable
investigation.
A. Determination of the level of gonadotropin
B. Progesteron test
C. Determination of the contents of testosteron-depotum in blood serum.
D. Computer tomography of the head
E. Ultrasound of organs of a small pelvis
17. In the woman of 24 years about earlier normal menstrual function, cycles
became irregular, according to tests of function diagnostics - anovulatory. The
contents of prolactin in blood is boosted. Choose the most suitable
investigation:
A. Computer tomography of the head
B. USI of organs of small pelvis
C. Determination of the contents of testosteron-depotum in blood serum
D. Progesterone assay
E. Determination of the level of gonadotropins
18. A woman in her 39th week of pregnancy, the second labour, has regular birth
activity. Uterine contractions take place every 3 minutes. What criteria
describe the beginning of the II labor stage the most precisely?
A. Cervical dilatation by no less than 4 cm
B. Cervical smoothing over 90%
C. Presenting part is in the lower region of small pelvis
D. Rupture of fetal bladder
E. Duration of uterine contractions over 30 seconds
19. A 20-year-old woman is having timed labor continued for 4 hours. Light
amniotic fluid came off. The fetus head is pressed to the orifice in the small
pelvis. The anticipated fetus mass is 4000,0 gpm 200,0 g. Heartbeat of the
fetus is normal. Intrinsic examination: cervix is absent, disclosure – 2 cm, the
fetal membranes are not present. The head is in 1-st plane of the pelvis, a
sagittal suture is in the left slanting dimension. What is the purpose of
glucose-calcium-hormone - vitaminized background conduction?
A. Prophylaxes of weakness of labor activity
B. Fetus hypoxia prophylaxes
C. Treatment of weakness of labor activity.
D. Antenatal preparation
E. Labor stimulation
20.A 24 years old primipara was hospitalised with complaints about discharge of
the amniotic waters. The uterus is tonic on palpation. The position of the fetus
is longitudinal, it is pressed with the head to pelvic outlet. Palpitation of the
fetus is rhythmical, 140 bpm, auscultated on the left below the navel. Internal
examination: cervix of the uterus is 2,5 cm long, dense, the external os is
closed, light amniotic waters out of it. Point a correct component of the
diagnosis:
A. Antenatal discharge of the amniotic waters
B. Early discharge of the amniotic waters
C. The end of the 1st stage of labour
D. Pathological preterm labour
E. The beginning of the 1st stage of labour
21. A 34 y.o. woman in her 29-th week of pregnancy, that is her 4-th labor to
come, was admitted to the obstetric department with complaints of sudden
and painful bloody discharges from vagina that appeared 2 hours ago. The
discharges are profuse and contain grumes. Cardiac funnction of the fetus is
rhytmic, 150 strokes in the minute, uterus tone is normal. The most probable
provisional diagnosis will be:
A. Placental presentation
B. Disseminated intravascular coagulation syndrome
C. Vasa previa
D. Detachment of normally located placenta
E. Bloody discharges
22.A 29 year old patient underwent surgical treatment because of the benign
serous epithelial tumour of an ovary. The postoperative period has elapsed
without complications. What is it necessary toprescribe for the
rehabilitational period:
A. Hormonotherapy and proteolytic enzymes
B. Magnitotherapy and vitamin therapy
C. The patient does not require further care
D. Antibacterial therapy and adaptogens
E. Lasertherapy and enzymotherapy
23.A 34-year-old woman with 10-week pregnancy (the second pregnancy) has
consulted gynaecologist to make a record in patient chart. There was a
hydramnion previous pregnancy, the birth weight of a child was 4086 g. What
tests are necessary first of all?
A. The test for tolerance to glucose
B. Fetus cardiophonography
C. Ultrasound of the fetus
D. Determination of the contents of alpha fetoprotein
E. Bacteriological test of discharge from the vagina
24.A 26 y.o. woman complains of sudden pains in the bottom of abdomen
irradiating to the anus, nausea, giddiness, bloody dark discharges from sexual
tracts for one week, the delay of menses for 4 weeks. Signs of the peritoneum
irritation are positive. Bimanual examination: borders of the uterus body and
its appendages are not determined because of sharp painfullness. The
diverticulum and painfullness of the back and dextral fornixes of the vagina
are evident. What is the most probable diagnosis?
A. Broken tubal pregnancy
B. Acute right-side adnexitis
C. Apoplexy of the ovary
D. Torsion of the crus of the ovary tumour
E. Acute appendicitis
25.At the gynaecological department there is a patient of 32 years with the
diagnosis: "acute bartholinitis". Body temperature is 38,2°C, leucocytes count
10,4×109/L, the ESR is 24 mm/hour. In the area of big gland of the
vestibulum - the dermahemia, the sign of the fluctuation, sharp tenderness
(pain). What is the most correct tactics of the doctor?
A. Surgical dissecting, a drainage of an abscess of the gland,
antibiotics
B. Antibiotics, Sulfanilamidums
C. Antibiotic therapy
D. Antibiotics, detoxication and biostimulants.
E. Surgical dissection, drainage of the abscess of the gland
26.A primagravida with pregnancy of 37-38 weeks complains of headache,
nausea, pain in epigastrium. Objective: the skin is acyanotic. Face is hydropic,
there is short fibrillar twitching of blepharons, muscles of the face and the
inferior extremities. The look is fixed. AP- 200/110 mm Hg; sphygmus of 92
bpm, intense. Respiration rate is 32/min. Heart activity is rhythmical.
Appreciable edemata of the inferior extremities are present. Urine is cloudy.
What medication should be administered?
A. Droperidolum of 0,25% - 2,0 ml
B. Dibazolum of 1% - 6,0 ml
C. Hexenalum of 1% - 2,0 ml
D. Pentaminum of 5% - 4,0 ml
E. Papaverine hydrochloride of 2% - 4,0 ml
27. An onset of severe preeclampsia at 16 weeks gestation might be caused by:
A. Hydatidiform mole
B. Anencephaly
C. Maternal renal disease
D. Interventricular defect of the fetus
E. Twin gestation
28.A woman had the rise of temperature up to 39°C on the first day after labour.
The rupture of fetal membranes took place 36 hours before labour. The
investigation of the bacterial flora of cervix of the uterus revealed
hemocatheretic streptococcus of group A. The uterus body is soft, tender.
Discharges are bloody, mixed with pus. Specify the most probable postnatal
complication:
A. Metroendometritis
B. Infection of the urinary system
C. Apostatis of junctures after the episiotomy
D. Thrombophlebitis of pelvic veins
E. Infected hematoma
29.A 24 y.o. patient 13 months after the first labour consulted a doctor about
amenorrhea. Pregnancy has concluded by a Cesarean section concerning to a
premature detachment of normally posed placenta hemorrhage has made low
fidelity 2000 ml owing to breakdown of coagulability of blood. Choose the
most suitable investigation:
A. Determination of the level of Gonadotropins
B. Progesteron assay
C. USI of organs of a small pelvis
D. Computer tomography of the head
E. Determination of the contents of Testosteron-Depotum in Serum of blood
30.A 34 year old woman in the 10th week of gestation (the second pregnancy)
consulted a doctor of antenatal clinic in order to be registered there. In the
previous pregnancy hydramnion was observed, the childs birth weight was
4086 g. What examination method should be applied in the first place?
A. The test for tolerance to glucose
B. US of fetus
C. Bacteriological examination of discharges from vagina
D. Determination of the contents of fetoproteinum
E. A cardiophonography of fetus
31. A 10 y.o. boy was ill with angina 2 weeks ago, has complaints of joint pain and
stiffness of his left knee and right elbow. There was fever (38,50) and ankle
disfunction, enlargement of cardiac dullness by 2 cm, tachycardia, weakness
of the 1st sound, gallop rhythm, weak systolic murmur near apex. What
diagnosis corresponds with such symptoms?
A. Acute rheumatic fever
B. Reiters disease
C. Reactive arthritis
D. Systemic lupus erythematosis
E. Juvenile rheumatoid arthritis
32.The disease began acutely. The frequent watery stool developed 6 hours ago.
The bodys temperature is normal. Then the vomiting was joined. On
examination: his voice is hoarse, eyes are deeply sunken in the orbits. The
pulse is frequent. Blood pressure is low. There is no urine. What is the
preliminary diagnosis?
A. Cholera
B. Typhoid fever
C. Salmonellosis
D. Toxic food-borne infection
E. Dysentery
33.At term of a gestation of 40 weeks height of standing of a uterine fundus is less
then assumed for the given term. The woman has given birth to the child in
weight of 2500 g, a length of a body 53 cm, with an assessment on a scale of
Apgar of 4-6 points. Labor were fast. The cause of such state of the child were:
A. Chronic fetoplacental insufficiency
B. Delay of an intra-uterine fetation
C. Infection of a fetus
D. Prematurity
E. Placental detachment
34.A pregnant woman may be diagnosed with hepatitis if it is confirmed by the
presence of elevated:
A. SGOT (ALT)
B. WBCs
C. Sedimentation rates
D. Alkaline phosphatase
E. BUN
35.A woman, aged 40, primigravida, with infertility in the medical history, on the
42-43 week of pregnancy. Labour activity is weak. Longitudinal presentation
of the fetus, I position, anterior position. The head of the fetus is engaged to
pelvic inlet. Fetus heart rate is 140 bmp, rhythmic, muffled. Cervix dilation is
4 cm. On amnioscopy: greenish colour of amniotic fluid and fetal membranes.
Cranial bones are dense, cranial sutures and small fontanel are diminished.
What should be tactics of delivery?
A. Caesarean section
B. Medication sleep, amniotomy, labour stimulation
C. Fetal hypoxia treatment, in the II period - forceps delivery
D. Amniotomy, labour stimulation, fetal hypoxia treatment
E. Fetal hypoxia treatment, conservative delivery
36.An endometrial adenocarcinoma that has extended tothe uterine serosa
would be classified as stage:
A. IIIA
B. IIA
C. IC
D. IIB
E. IVAB
37. Which of the methods of examination is the most informative in the
diagnostics of a tube infertility?
A. Laparoscopy with chromosalpingoscopy
B. Hysterosalpingography
C. Pertubation
D. Transvaginal echography
E. Bicontrast pelviography
38.A pregnant woman (35 weeks), aged 25, was admitted to the hospital because
of bloody discharges. In her medical history there were two artificial
abortions. In a period of 28-32 weeks there was noted the onset of
hemorrhage and USD showed a placental presentation. The uterus is in
normotonus, the fetus position is transversal (Ist position). The heartbeats is
clear, rhythmical, 140 bpm. What is the further tactics of the pregnant woman
care?
A. To perform a delivery by means of Cesarean section
B. Tointroduct the drugs to increase the blood coagulation and continue
observation
C. Tokeep the intensity of hemorrhage under observation and after the bleeding
is controlled to prolong the pregnancy
D. Stimulate the delivery by intravenous introduction of oxytocin
E. Toperform the hemotransfusion and to prolong the pregnancy
39.A woman, primagravida, consults a gynecologist on 05.03.2012. A week ago
she felt the fetus movements for the first time. Last menstruation was on
10.01.2012. When should she be given maternity leave?
A. 8 August
B. 22 August
C. 5 September
D. 11 July
E. 25 July
40.Condition of a parturient woman has been good for 2 hours after live birth:
uterus is thick, globe-shaped, its bottom is at the level of umbilicus, bleeding
is absent. The clamp put on the umbilical cord remains at the same level,
when the woman takes a deep breath or she is being pressed over the
symphysis with the verge of hand, the umbilical cord drows into the vagina.
Bloody discharges from the sexual tracts are absent. What is the doctors
further tactics?
A. To do manual removal of afterbirth
B. Todo curettage of uterine cavity
C. Tointroduct oxitocine intravenously
D. Toapply Abduladze method
E. Toapply Credes method
41. The woman who has delivered twins has early postnatal hypotonic uterine
bleeding reached 1,5% of her bodyweight. The bleeding is going on.
Conservative methods to arrest the bleeding have been found ineffective. The
conditions of patient are pale skin, acrocyanosis, oliguria. The woman is
confused. The pulse is 130 bpm, BP– 75/50 mm Hg. What is the further
treatment?
A. Uterine extirpation
B. Uterine vessels ligation
C. Putting clamps on the uterine cervix
D. Inner glomal artery ligation
E. Supravaginal uterine amputation
42.A 26 y.o. woman complains of a mild bloody discharge from the vagina and
pain in the lower abdomen. She has had the last menstruation 3,5 months ago.
The pulse is 80 bpm. The blood pressure (BP) is 110/60 mm Hg and body
temperature is 36,6°C. The abdomen is tender in the lower parts. The uterus is
enlarged up to 12 weeks of gestation. What is your diagnosis?
A. Inevitable abortion
B. Incomplete abortion
C. Disfunctional bleeding
D. Complete abortion
E. Incipient abortion
43.18 y.o. woman complains of pain in the lower abdomen. Some minutes before
she has suddenly appeared unconscious at home. The patient had no menses
within last 3 months. On examination: pale skin, the pulse- 110 bpm, BP-
80/60 mm Hg. The Schyotkins sign is positive. Hb- 76 g/L. The vaginal
examination: the uterus is a little bit enlarged, its displacement is painful.
There is also any lateral swelling of indistinct size. The posterior fornix of the
vagina is tendern and overhangs inside. What is the most probable diagnosis?
A. Impaired extrauterine pregnancy
B. Acute appendicitis
C. Twist of cystoma of right uterine adnexa
D. Ovarian apoplexy
E. Acute salpingoophoritis
44.A 20 y.o. pregnant woman with 36 weeks of gestation was admitted to the
obstetrical hospital with complains of pain in the lower abdomen and bloody
vaginal discharge. The general condition of the patient is good. Her blood
pressure is 120/80 mm Hg. The heart rate of the fetus is 140 bpm, rhythmic.
Vaginal examination: the cervix of the uterus is formed and closed. The
discharge from vagina is bloody up to 200 ml per day. The head of the fetus is
located high above the minor pelvis entry. A soft formation was defined
through the anterior fornix of the vagina. What is the probable diagnosis?
A. Placental presentation
B. Premature placental separation
C. Threatened premature labor
D. Incipient abortion
E. Uterine rupture
45.In the gynecologic office a 28 y.o. woman complains of sterility within three
years. The menstrual function is not impaired. There were one artificial
abortion and chronic salpingo-oophoritis in her case history. Oral
contraceptives were not used. Her husbands analysis of semen is without
pathology. What diagnostic method will you start from the workup in this case
of sterility?
A. Hysterosalpingography
B. Ultra sound investigation
C. Hysteroscopia
D. Diagnostic scraping out of the uterine cavity
E. Hormone investigation
46.A 28-year-old patient underwent endometrectomy as a result of incomplete
abortion. Blood loss was at the rate of 900 ml. It was necessary tostart
hemotransfusion. After transfusion of 60 ml of erythrocytic mass the patient
presented with lumbar pain and fever which resulted in hemotransfusion
stoppage. 20 minutes later the patients condition got worse: she developed
adynamia, apparent skin pallor, acrocyanosis, profuse perspiration. to-
38,5°C, Ps- 110/min, AP- 70/40 mm Hg. What is the most likely diagnosis?
A. Hemotransfusion shock
B. DIC syndrome
C. Septic shock
D. Hemorrhagic shock
E. Anaphylactic shock
47. A 58-year-old female patient came to the antenatal clinic complaining of
bloody light-red discharges from the genital tracts. Menopause is 12 years.
Gynaecological examination revealed age involution of externalia and vagina;
uterine cervix was unchanged, there were scant bloody discharges from
uterine cervix, uterus was of normal size; uterine appendages were not
palpable; parametria were free. What is the most likely diagnosis?
A. Uterine carcinoma
B. Cervical carcinoma
C. Granulosa cell tumor of ovary
D. Atrophic colpitis
E. Abnormalities of menstrual cycle of climacteric nature
48.The results of a separate diagnostic curettage of the mucous of the uterus
cervix and body made up in connection with bleeding in a postmenopausal
period: the scrape of the mucous of the cervical canal revealed no pathology,
in endometrium - the highly differentiated adenocarcinoma was found.
Metastases are not found. What method of treatment is the most correct?
A. Surgical treatment and hormonotherapy
B. Surgical treatment and radial therapy
C. Surgical treatment + chemotherapy
D. Radial therapy
49.A 27 y.o. woman complains of having the disorders of menstrual function for 3
months, irregular pains in abdomen. On bimanual examination: in the dextral
appendage range of uterus there is an elastic spherical formation, painless, 7
cm in diameter. USI: in the right ovary - a fluid formation, 4 cm in diameter,
unicameral, smooth. What method of treatment is the most preferable?
A. Prescription of an estrogen-gestogen complex for 3 months with
repeated examination
B. Dispensary observation of the patient
C. Chemotherapeutic treatment
D. Anti-inflammatory therapy
E. Operative treatment
50.A 40 year old patient complains of yellowish discharges from the vagina.
Bimanual examination revealed no pathological changes. The smear contains
Trichomonas vaginalis and blended flora. Colposcopy revealed two hazy fields
on the frontal labium, with a negative Iodine test. Your tactics:
A. Treatment of specific colpitis and subsequent biopsy
B. Cervix ectomy
C. Cryolysis of cervix of the uterus
D. Diathermocoagulation of the cervix of the uterus
E. Specific treatment of Trichomonas colpitis
51. A 26-year-old secundipara at 40 weeks of gestation arrived at the maternity
ward after the beginning of labor activity. 2 hours before, bursting of waters
occurred. The fetus was in a longitudinal lie with cephalic presentation.
Abdominal circumference was 100 cm, fundal height - 42 cm. Contractions
occurred every 4-5 minutes and lasted 25 seconds each. Internal obstetric
examination revealed cervical effacement, opening by 4 cm. Fetal bladder was
absent. Fetal head was pressed against the pelvic inlet. What complication
arose in childbirth?
A. Early amniorrhea
B. Secondary uterine inertia
C. Clinically narrow pelvis
D. Discoordinated labor
E. Primary uterine inertia
52.A 28-year-old parturient complains about headache, vision impairment,
psychic inhibition. Objectively: AP- 200/110mm Hg, evident edemata of legs
and anterior abdominal wall. Fetus head is in the area of small pelvis. Fetal
heartbeats is clear, rhythmic, 190/min. Internal examination revealed
complete cervical dilatation, fetus head was in the area of small pelvis. What
tactics of labor management should be chosen?
A. Forceps operation
B. Cesarean
C. Conservative labor management with episiotomy
D. Stimulation of labor activity
E. Embryotomy
53.A 28 year old woman had the second labour and born a girl with
manifestations of anemia and progressing jaundice. The childs weight was 3
400 g, the length was 52 cm. The womans blood group is B (III) Rh-, the
fathers blood group is A (III) Rh+, the childs blood group is B (III) Rh+. What
is the cause of anemia?
A. Rhesus incompatibility
B. Antigen A incompatibility
C. Antigen AB incompatibility
D. Intrauterine infection
E. Antigen B incompatibility
54.A 48 year old female patient complains about contact haemorrhage. Speculum
examination revealed hypertrophy of uterus cervix. It resembles of
cauliflower, it is dense and can be easily injured. Bimanual examination
revealed that fornices were shortened, uterine body was nonmobile. What is
the most probable diagnosis?
A. Cervical carcinoma
B. Metrofibroma
C. Cervical pregnancy
D. Cervical papillomatosis
E. Endometriosis
55.A 37 y.o. primigravida woman has been having labor activity for 10 hours.
Labor pains last for 20-25 seconds every 6-7 minutes. The fetus lies in
longitude, presentation is cephalic, head is pressed upon the entrance to the
small pelvis. Vaginal examination results: cervix of uterus is up to 1 cm long,
lets 2 transverse fingers in. Fetal bladder is absent. What is the most probable
diagnosis?
A. Primary uterine inertia
B. Discoordinated labor activity
C. Pathological preliminary period
D. Secondary uterine inertia
E. Normal labor activity
56.Laparotomy was performed to a 54 y.o. woman on account of big formation in
pelvis that turned out to be one-sided ovarian tumor along with considerable
omental metastases. The most appropriate intraoperative tactics involves:
A. Ablation of omentum, uterus and both ovaries with tubes
B. Biopsy of omentum
C. Ablation of an ovary and omental metastases
D. Ablation of omentum and both ovaries with tubes
E. Biopsy of an ovary
57. A parturient complains about pain in the mammary gland. Palpation revealed
a 3×4 cm large infiltration, soft in the centre. Body temperature is 38,5°C.
What is the most probable diagnosis?
A. Acute purulent mastitis
B. Pleuritis
C. Birth trauma
D. Retention of milk
E. Pneumonia
58.A 43 y.o. patient complains of formation and pain in the right mammary
gland, rise of temperature up to 37,2°C during the last 3 months. Condition
worsens before the menstruation. On examination: edema of the right breast,
hyperemia, retracted nipple. Unclear painful infiltration is palpated in the
lower quadrants. What is the most probable diagnosis?
A. Cancer of the right mammary gland
B. Tuberculosis of the right mammary gland
C. Right-side chronic mastitis
D. Right-side acute mastitis
E. Premenstrual syndrome
59.A 14 year old girl complains of profuse bloody discharges from genital tracts
during 10 days after suppresion of menses for 1,5 month. Similiar bleedings
recur since 12 years on the background of disordered menstrual cycle. On
rectal examination: no pathology of the internal genitalia. In blood: Hb - 70
g/l, RBC- 2,3×1012/l, Ht - 20. What is the most probable diagnosis?
A. Juvenile bleeding, posthemorrhagic anemia
B. Polycyst ovarian syndrome
C. Werlholfs disease
D. Hormonoproductive ovary tumor
E. Incomplete spontaneous abortion
60.A 33-year-old woman was urgently brought to clinic with complaints of the
pain in the lower part of the abdomen, mostly on the right, irradiating to
rectum, she also felt dizzy. The above mentioned complaints developed
acutely at night. Last menses were 2 weeks ago. On physical exam: the skin is
pale, Ps - 92 bpm, t- 36,6°C, BP- 100/60 mm Hg. The abdomen is tense,
slightly tender in lower parts, peritoneal symptoms are slightly positive. Hb-
98 g/L. What is the most probable diagnosis?
A. Apoplexy of the ovary
B. Renal colic
C. Intestinal obstruction
D. Acute appendicitis
E. Abdominal pregnancy
61. A secundipara has regular birth activity. Three years agoshe had cesarean
section for the reason of acute intrauterine hypoxia. During parodynia she
complains of extended pain in the area of postsurgical scar. Objectively: fetus
pulse is rhythmic - 140 bpm. Vaginal examination shows 5 cm cervical
dilatation. Fetal bladder is intact. What is the tactics of choice?
A. Cesarean section
B. Obstetrical forceps
C. Augmentation of labour
D. Waiting tactics of labor management
E. Vaginal delivery
62.A 54-year-old female patient consulted a doctor about bloody discharges from
the genital tracts after 2 years of amenorrhea. USI and bimanual examination
revealed no genital pathology. What is the tactics of choice?
A. Fractional biopsy of lining of uterus and uterine mucous
membranes
B. Contracting drugs
C. Hysterectomy
D. Estrogenic haemostasia
E. Styptic drugs
63.Examination of a just born placenta reveals defect 2x3 cm large. Hemorrhage
is absent. What tactic is the most reasonable?
A. Manual uretus cavity revision
B. Prescription of uterotonic medicines
C. Parturient supervision
D. Instrumental uterus cavity revision
E. External uterus massage
64.A 27 y.o. gravida with 17 weeks of gestation was admitted to the hospital.
There was a history of 2 spontaneous miscarriages. On bimanual examination:
uterus is enlarged to 17 weeks of gestation, uterus cervix is shortened, isthmus
allows to pass the finger tip. The diagnosis is isthmico-cervical insufficiency.
What is the doctors tactics?
A. To place suture on the uterus cervix
B. Toperform amniocentesis
C. Tointerrupt pregnancy
D. Toadminister tocolytic therapy
E. Toadminister hormonal treatment
65.A 27-year-old woman presents at the maternity welfare centre because of
infertility. She has had sexual life in marriage for 4 years, doesnt use
contraceptives. She hasnt get pregnant. On examination: genital development
is without pathology, uterine tubes are passable, basal (rectal) temperature is
one-phase during last 3 menstrual cycles. What is the infertility cause?
A. Anovular menstrual cycle
B. Abnormalities in genital development
C. Chronic adnexitis
D. Immunologic infertility
E. Genital endometriosis
66.A 43 y.o. woman complains of contact hemorrhages during the last 6 months.
Bimanual examination: cervix of the uterus is enlarged, its mobility is
reduced. Mirrors showed the following: cervix of the uterus is in the form of
cauliflower. Chrobak and Schiller tests are positive. What is the most probable
diagnosis?
A. Cancer of cervix of the uterus
B. Cervical pregnancy
C. Leukoplakia
D. Nascent fibroid
E. Polypus of the cervis of the uterus
67. A 26-year-old woman gave birth to a child 6 months ago. She applied to
gynecologist complaining of menstruation absence. The child is breast-fed.
Vagina exam: uterus is of normal form, dense consistence. What is the most
probable diagnosis?
A. Physiological amenorrhea
B. Sheehans syndrome
C. Gestation
D. Pseudoamenorrhea
E. Ashermans syndrome
68.A primagravida in her 20th week of gestation complains about pain in her
lower abdomen, blood smears from the genital tracts. The uterus has an
increased tonus, the patient feels the fetus movements. Bimanual examination
revealed that the uterus size corresponded the term of gestation, the uterine
cervix was contracted down to 0,5 cm, the external orifice was open by 2 cm.
The discharges were bloody and smeary. What is the most likely diagnosis?
A. Incipient abortion
B. Abortion in progress
C. Missed miscarriage
D. Incomplete abortion
E. Risk of abortion
69.Full-term pregnancy. Body weight of the pregnant woman is 62 kg. The fetus
has the longitudinal position, the fetal head is pressed against the pelvic inlet.
Abdominal circumference is 100 cm. Fundal height is 35 cm. What is the
approximate weight of the fetus?
A. 3 kg 500 g
B. 4 kg
C. 3 kg
D. 4 kg 500 g
E. 2 kg 500 g
70.A patient was admitted to the hospital with complaints of periodical pain in
the lower part of abdomen that gets worse during menses, weakness, malaise,
nervousness, dark bloody smears from vagina directly before and after
menses. Bimanual examination revealed that uterus body is enlarged,
appendages cannot be palpated, posterior fornix has tuberous surface.
Laparoscopy revealed: ovaries, peritoneum of rectouterine pouch and
pararectal fat have "cyanotic eyes". What is the most probable diagnosis?
A. Disseminated form of endometriosis
B. Chronic salpingitis
C. Ovarian cystoma
D. Tuberculosis of genital organs
E. Polycystic ovaries
71. A gravida with 7 weeks of gestation is referred for the artificial abortion. On
operation while dilating cervical canal with Hegar dilator No.8 a doctor
suspected uterus perforation. What is immediate doctors tactics to confirm
the diagnosis?
A. Probing of uterus cavity
B. Ultrasound examination
C. Bimanual examination
D. Laparoscopy
E. Metrosalpingography
72. A pregnant woman in her 8th week was admitted to the hospital for artificial
abortion. In course of operation during dilatation of cervical canal of uterus by
means of Hegars dilator No.8 the doctor suspected uterus perforation. What is
the immediate tactics for confirmation of this diagnosis?
A. Uterine probing
B. US examination
C. Metrosalpingography
D. Laparoscopy
E. Bimanual examination
73. A 59 year old female patient applied to a maternity welfare clinic and
complained about bloody discharges from the genital tracts. Postmenopause is
12 years. Vaginal examination revealed that external genital organs had signs
of age involution, uterus cervix was not erosive, small amount of bloody
discharges came from the cervical canal. Uterus was of normal size, uterine
appendages were unpalpable. Fornices were deep and painless. What method
should be applied for the diagnosis specification?
A. Separated diagnosic curretage
B. Culdoscopy
C. Puncture of abdominal cavity through posterior vaginal fornix
D. Laparoscopy
E. Extensive colposcopy
74. A 25-year-old woman complains of profuse foamy vaginal discharges, foul,
burning and itching in genitalia region. She has been ill for a week.
Extramarital sexual life. On examination: hyperemia of vaginal mucous,
bleeding on touching, foamy leucorrhea in the urethral area. What is the most
probable diagnosis?
A. Trichomonas colpitic
B. Bacterial vaginosis
C. Chlamydiosis
D. Gonorrhea
E. Vagina candidomicosis
75. A 26 year old woman who delivered a child 7 months ago has been suffering
from nausea, morning vomiting, sleepiness for the last 2 weeks. She suckles
the child, menstruation is absent. She hasnt applied any contraceptives. What
method should be applied in order to specify her diagnosis?
A. Ultrasonic examination
B. Speculum examination
C. Palpation of mammary glands and pressing-out of colostrum
D. Roentgenography of small pelvis organs
E. Bimanual vaginal examination
76. A newborns head is of dolichocephalic shape, that is front-to-back elongated.
Examination of the occipital region revealed a labour tumour located in the
middle between the prefontanel and posterior fontanel. Specify the type of
fetal presentation:
A. Posterior vertex presentation
B. Presentation of the bregma
C. Face presentation
D. Brow presentation
E. Anterior vertex presentation
77. A woman consulted a doctor on the 14th day after labour about sudden pain,
hyperemy and induration of the left mammary gland, body temperature rise
up to 39°C, headache, indisposition. Objectively: fissure of nipple,
enlargement of the left mammary gland, pain on palpation. What pathology
would you think about in this case?
A. Lactational mastitis
B. Phlegmon of mammary gland
C. Fibrous adenoma of the left mammary gland
D. Lacteal cyst with suppuration
E. Breast cancer
78. A young woman applied to gynecologist due to her pregnancy of 4-5 weeks.
The pregnancy is desirable. Anamnesis stated that she had rheumatism in the
childhood. Now she has combined mitral heart disease with the priority of
mitral valve deficiency. When will she need the inpatient treatment (what
periods of pregnancy)?
A. 8-12 weeks, 28–32 weeks, 37 weeks
B. 12-16 weeks, 27-28 weeks, 37-38 weeks
C. 16 weeks, 34 weeks, 39-40 weeks
D. 6-7weeks, 16 weeks, 38 weeks
E. 10-12 weeks, 24 weeks, 37-38 weeks
79. A woman in the first half of pregnancy was brought to clinic by an ambulance.
Term of pregnancy is 36 weeks. She complains of intensive pain in the
epigastrium, had vomiting for 2 times. Pain started after the patient had eaten
vinaigrette. Swelling of lower extremities. BP - 140/100 mm Hg. Urine became
curd after boiling. What is the most probable diagnosis?
A. Preeclampsia
B. Food toxicoinfection
C. Exacerbation of pyelonephritis
D. Dropsy of pregnant women
E. Nephropathy of the 3rd degree
80.A 13 year old girl consulted the school doctor on account of moderate bloody
discharge from the genital tracts, which appeared 2 days ago. Secondary
sexual characters are developed. What is the most probable cause of bloody
discharge?
A. Menarche
B. Werlhofs disease
C. Haemophilia
D. Juvenile hemorrhage
E. Endometrium cancer
81. In 10 min after childbirth by a 22-year-old woman, the placenta was
spontaneousely delivered and 100 ml of blood came out. Woman weight - 80
kg, infant weight - 4100 g, length - 53 cm. The uterus contracted. In 10
minutes the hemorrhage renewed and the amount of blood constitued 300 ml.
What amount of blood loss is permissible for this woman?
A. 400 ml
B. 650 ml
C. 300 ml
D. 1000 ml
E. 500 ml
82.A pregnant woman was registered in a maternity welfare clinic in her 11th
week of pregnancy. She was being under observation during the whole term,
the pregnancy course was normal. What document must the doctor give the
pregnant woman to authorize her hospitalization in maternity hospital?
A. Exchange card
B. Medical certificate
C. Sanitary certificate
D. Appointment card for hospitalization
E. Individual prenatal record
83.After examination a 46-year-old patient was diagnosed with left breast cancer
T2N2M0, cl. gr. II-a. What will be the treatment plan for this patient?
A. Radiation therapy + operation + chemotherapy
B. Operation only
C. Radiation therapy only
D. Chemotherapy only
E. Operation + radiation therapy
84.Immediately after delivery a woman had haemorrhage, blood loss exceeded
postpartum haemorrhage rate and was progressing. There were no symptoms
of placenta detachment. What tactics should be chosen?
A. Manual removal of placenta and afterbirth
B. Instrumental revision of uterine cavity walls
C. Intravenous injection of methylergometrine with glucose
D. Removal of afterbirth by Credes method
E. Uterus tamponade
85.A 30 y.o. primigravida woman has got intensive labor pain every 1-2 minutes
that lasts 50 seconds. The disengagement has started. The perineum with the
height of 4 cm has grown pale. What actions are necessary in this situation?
A. Episiotomy
B. Expectant management
C. Perineotomy
D. Perineum protection
E. Vacuum extraction of fetus
86.A 30-year-old gravida consulted a gynecologist about bright red bloody
discharges from the vagina in the 32 week of gestation. She was hospitalized
with a suspicion of placental presentation. Under what conditions is it rational
to conduct the internal examination in order to make a diagnosis?
A. In the operating room prepared for the operation
B. In the admission ward of maternity hospital
C. The examination is not to be conducted because of risk of profuse
haemorrhage
D. In the delivery room keeping to all the aseptics regulations
E. In the examination room of antenatal clinic
87. A 28 y.o. primagravida, pregnancy is 15-16 weaks of gestation, presents to the
maternity clinics with dull pain in the lower part of the abdomen and in
lumbar area. On vaginal examination: uterus cervix is 2,5 cm, external
isthmus allows to pass the finger tip. Uterus body is enlarged according to the
pregnancy term. Genital discharges are mucous, mild. What is the diagnosis?
A. Threatened spontaneous abortion
B. Hydatid molar pregnancy
C. Placenta presentation
D. Spontaneous abortion which has begun
E. Stopped pregnancy
88.A primapara with pelvis size 25-28-31-20 cm has active labor activity. Waters
poured out, clear. Fetus weight is 4500 g, the head is engaged to the small
pelvis inlet. Vastens sign as positive. Cervix of uterus is fully dilated. Amniotic
sac is absent. The fetus heartbeat is clear, rhythmic, 136 bpm. What is the
labor tactics?
A. Caesarean section
B. Obstetrical forseps
C. Vacuum extraction of the fetus
D. Conservative tactics of labor
E. Stimulation of the labor activity
89.Internal obstetric examination of a parturient woman revealed that the
sacrum hollow was totally occupied with fetus head, ischiadic spines couldnt
be detected. Sagittal suture is in the straight diameter, occipital fontanel is
directed towards symphysis. In what plane of small pelvis is the presenting
part of the fetus?
A. Plane of pelvic outlet
B. Wide pelvic plane
C. Plane of pelvic inlet
D. Over the pelvic inlet
E. Narrow pelvic plane
90.A 30 y.o. woman has the 2-nd labour that has been lasting for 14 hours.
Hearbeat of fetus is muffled, arrhythmic, 100/min. Vaginal examination:
cervix of uterus is completely opened, fetus head is level with outlet from
small pelvis. Saggital suture is in the straight diameter, small crown is near
symphysis. What is the further tactics of handling the delivery?
A. Use of obstetrical forceps
B. Use of cavity forceps
C. Cesarean section
D. Stimulation of labour activity by oxytocin
E. Cranio-cutaneous (Ivanovs) forceps
91. During examination of a patient, masses in the form of condyloma on a broad
basis are found in the area of the perineum. What is the tactics of the doctor?
A. To send a woman into dermatological and venerological centre
B. Cryodestruction of condyloms
C. Chemical coagulator treatment
D. Antiviral treatment
E. Surgical ablation of condyloms
92.A woman at 30 weeks pregnant has had an attack of eclampsia at home. On
admission to the maternity ward AP is 150/100 mm Hg. Predicted fetal weight
is 1500 g. There is face and shin pastosity. Urine protein is 0,66o/oo.
Parturient canal is not ready for delivery. An intensive complex therapy has
been started. What is the correct tactics of this case management?
A. Delivery by cesarean section
B. Continue therapy and prolong pregnancy for 3-4 weeks
C. Treat preeclampsia and achieve the delivery by way of conservative
management
D. Labor induction by intravenous oxytocin or prostaglandins
E. Continue therapy and prolong pregnancy for 1-2 weeks
93.A 28 year old woman has bursting pain in the lower abdomen during
menstruation; chocolate-like discharges from vagina. It is known from the
anamnesis that the patient suffers from chronic adnexitis. Bimanual
examination revealed a tumour-like formation of heterogenous consistency
7*7 cm large to the left from the uterus. The formation is restrictedly movable,
painful when moved. What is the most probable diagnosis?
A. Endometrioid cyst of the left ovary
B. Fibromatous node
C. Tumour of sigmoid colon
D. Exacerbation of chronic adnexitis
E. Follicular cyst of the left ovary
94.Vaginal inspection of a parturient woman revealed: cervix dilation is up to 2
cm, fetal bladder is intact. Sacral cavity is free, sacral promontory is reachable
only with a bent finger, the inner surface of the sacrococcygeal joint is
accessible for examination. The fetus has cephalic presentation. Sagittal
suture occupies the transverse diameter of pelvic inlet, the small fontanel to
the left, on the side. What labor stage is this?
A. Cervix dilatation stage
B. Prodromal stage
C. Placental stage
D. Stage of fetus expulsion
E. Preliminary stage
95.A 68-year-old patient consulted a doctor about a tumour in her left mammary
gland. Objectively: in the upper internal quadrant of the left mammary gland
there is a neoplasm up to 2,5 cm in diameter, dense, uneven, painless on
palpation. Regional lymph nodes are not enlarged. What is the most likely
diagnosis?
A. Cancer
B. Lipoma
C. Fibroadenoma
D. Cyst
E. Mastopathy
96.A 40-year-old female patient has been observing profuse menses accompanied
by spasmodic pain in the lower abdomen for a year. Bimanual examination
performed during menstruation revealed a dense formation up to 5 cm in
diameter in the cervical canal. Uterus is enlarged up to 5-6 weeks of
pregnancy, movable, painful, of normal consistency. Appendages are not
palpable. Bloody discharges are profuse. What is the most likely diagnosis?
A. Nascent submucous fibromatous node
B. Cervical myoma
C. Algodismenorrhea
D. Abortion in progress
E. Cervical carcinoma
97. A 29-year-old patient complains of sterility. Sexual life is for 4 years being
married, does not use contraception. There was no pregnancy before. On
physical examination, genitals are developed normally. Uterine tubes are
passable. Rectal temperature during three menstrual cycles is monophase.
What is the most probable reason for sterility?
A. Anovulatory menstrual cycle
B. Genital endometriosis
C. Anomalies of genitals development
D. Chronic adnexitis
E. Immunologic sterility
98.A 45 y.o. woman complains of contact bleedings during 5 months. On
speculum examination: hyperemia of uterus cervix, looks like cauliflower,
bleeds on probing. On bimanual examination: cervix is of densed consistensy,
uterus body isnt enlarged, mobile, nonpalpable adnexa, parametrium is free,
deep fornixes. What is the most likely diagnosis?
A. Cancer of cervix of uterus
B. Cancer of body of uterus
C. Cervical pregnancy
D. Polypose of cervix of uterus
E. Fibromatous node which is being born
99.10 minutes after delivery a woman discharged placenta with a tissue defect
5×6 cm large. Discharges from the genital tracts were profuse and bloody.
Uterus tonus was low, fundus of uterus was located below the navel.
Examination of genital tracts revealed that the uterine cervix, vaginal walls,
perineum were intact. There was uterine bleeding with following blood
coagulation. Your actions to stop the bleeding:
A. To make manual examination of uterine cavity
B. Toadminister uterotonics
C. Tointroduce an ether-soaked tampon into the posterior fornix
D. Toapply hemostatic forceps upon the uterine cervix
E. Toput an ice pack on the lower abdomen
100. On the 5th day after labor body temperature of a 24-year-old parturient
suddenly rose up to 38,7°C. She complains about weakness, headache,
abdominal pain, irritability. Objectively: AP- 120/70 mm Hg, Ps- 92 bpm, to-
38,7°C. Bimanual examination revealed that the uterus was enlarged up to 12
weeks of pregnancy, it was dense, slightly painful on palpation. Cervical canal
lets in 2 transverse fingers, discharges are moderate, turbid, with foul smell.
In blood: skeocytosis, lymphopenia, ESR - 30 mm/h. What is the most likely
diagnosis?
A. Endometritis
B. Metrophlebitis
C. Lochiometra
D. Parametritis
E. Pelviperitonitis
101. A 20 y.o. patient complains of amenorrhea. Objectively: hirsutism,
obesity with fat tissue prevailing on the face, neck, upper part of body. On the
face there are acne vulgaris, on the skin - striae cutis distense. Psychological
and intellectual development is normal. Gynecological condition: external
genitals are moderately hairy, acute vaginal and uterine hypoplasia. What
diagnosis is the most probable?
A. Itsenko-Cushing syndrome
B. Turners syndrome
C. Shichans syndrome
D. Babinski-Froehlich syndrome
E. Stein-Leventals syndrome
102. A 27 y.o. woman suffers from pyelonephritits of the only kidney. She
presents to the maternity welfare centre because of suppresion of menses for
2,5 months. On examination pregnancy 11 weeks of gestation was revealed. In
urine: albumine 3,3 g/L, leucocytes cover the field of vision. What is doctors
tactics in this case?
A. Immediate pregancy interruption
B. Pregnancy interruption at 24-25 weeks
C. Maintenance of pregnancy till delivery term
D. Pregnancy interruption after urine normalization
E. Maintenance of pregnancy till 36 weeks
103. An 18-year-old primigravida in her 27-28 week of gestation underwent
an operation on account of acute phlegmonous appendicitis. In the
postoperative period it is necessary to take measures for prevention of the
following pregnancy complication:
A. Noncarrying of pregnancy
B. Late gestosis
C. Fetus hypotrophy
D. Intestinal obstruction
E. Premature placenta detachment
104. A 24-year-old female patient complains of acute pain in the lower
abdomen that turned up after a physical stress. She presents with nausea,
vomiting, dry mouth and body temperature 36,6°C. She has a right ovarian
cyst in history. Bimanual examination reveals that uterus is dense, painless, of
normal size. The left fornix is deep, uterine appendages arent palpable, the
right fornix is contracted. There is a painful formation on the right of uterus.
Its round, elastic and mobile. It is 7×8 cm large. In blood: leukocytosis with
the left shit. What is the most likely diagnosis?
A. Ovarian cyst with pedicle torsion
B. Acute metritis
C. Extrauterine pregnancy
D. Right-sided pyosalpinx
E. Subserous fibromyoma of uterus
105. A parturient woman is 23 years old. Vaginal obstetric examination
reveals full cervical dilatation. There is no fetal bladder. Fetal head is in the
plane of pelvic outlet. Sagittal suture is in mesatipellic pelvis, anterior fontanel
is closer to pubes. The fetal head diameter in such presentation will be:
A. Suboccipito-bregmaticus
B. Suboccipitio-frontalis
C. Mento-occipitalis
D. Fronto-occipitalis recta
E. Biparietal
106. A pregnant 26-year-old woman was admitted to a hospital for
abdominal pain and bleeding from the genital tract. Bimanual examination
revealed that uterus was the size of 9 weeks of pregnancy, the cervical canal let
a finger through. Fetal tissues could be palpated in the orifice. There was
moderate vaginal bleeding. What is the tactics of choice?
A. Instrumental extraction of fetal tissue
B. Therapy for the maintenance of pregnancy
C. Administration of hormones
D. Surveillance
E. Hemostatic and antianemic therapy
107. A 42-year-old woman has had hyperpolymenorrhea and progressing
algodismenorrhea for the last 10 years. Gynaecological examination revealed
no changes of uterine cervix; discharges are moderate, of chocolate colour,
uterus is slightly enlarged and painful, appendages are not palpable, the
fornices are deep and painless. What is the most likely diagnosis?
A. Uterine endometriosis
B. Uterine carcinoma
C. Endomyometritis
D. Adnexal endmetriosis
E. Subserous uterine fibromyoma
108. On the tenth day after discharge from the maternity house a 2-year-old
patient consulted a doctor about body temperature rise up to 39°C, pain in the
right breast. Objectively: the mammary gland is enlarged, there is a
hyperemized area in the upper external quadrant, in the same place there is an
ill-defined induration, lactostasis, fluctuation is absent. Lymph nodes of the
right axillary region are enlarged and painful. What is the most likely
diagnosis?
A. Lactational mastitis
B. Erysipelas
C. Abscess
D. Dermatitis
E. Tumour
109. During the dynamic observation over a parturient woman in the second
stage of labor it was registered that the fetal heart rate fell down to 90-
100/min and didnt come to normal after contractions. Vaginal examination
revealed the complete cervical dilatation, the fetal head filling the entire
posterior surface of the pubic symphysis and sacral hollow; the sagittal suture
lied in the anteroposterior diameter of the pelvic outlet, the posterior
fontanelle was in front under the pubic arch. What plan for further labour
management should be recommended?
A. Application of forceps minor
B. Episiotomy
C. Caesarean section
D. Application of cavity forceps
E. Stimulation of labour activity by intravenous injection of oxytocin
110. A 27-year-old sexually active female complains of numerous vesicles on
the right sex lip, itch and burning. Eruptions regularly turn up before
menstruation and disappear 8-10 days later. What is the most likely
diagnosis?
A. Herpes simplex virus
B. Primary syphilis
C. Bartholinitis
D. Cytomegalovirus infection
E. Genital condylomata
111. A 36-year-old female pesented to a gynecological hospital with a
significant bleeding from the genital tract and a 1-month delay of
menstruation. Bimanual examination revealed soft barrel-shaped cervix.
Uterus was of normal size, somewhat softened. Appendages were
unremarkable on both sides. Speculum examination revealed that the cervix
was cyanotic, enlarged, with the the external orifice disclosed up to 0,5 cm.
Urine hCG test was positive. What is the most likely diagnosis?
A. Cervical pregnancy
B. Uterogestation
C. Threatened miscarriage
D. Ectopic pregnancy
E. Abortion in progress
112. A 26-year-old woman complains of having bloody discharges from the
genitals for the last 14 days, abdominal pain, general fatiguability, weakness,
weight loss, body temperature rise, chest pain, obstructed respiration. 5 weeks
ago she underwent induced abortion in the 6-7 week of gestation. Objectively:
the patient is pale and inert. Bimanual examination revealed that the uterus
was enlarges up to 8-9 weeks of gestation. In blood: Hb- 72 g/l. Urine test for
chorionic gonadotropin gave the positive result. What is the most likely
diagnosis?
A. Chorioepithelioma
B. Uterus perforation
C. Uterine carcinoma
D. Uterine fibromyoma
E. Metroendometritis
113. A 28-year-old patient complains of discomfort, acute pain in the lower
third of the left labia majora. The disease began suddenly after menstruation.
Objectively: body temperature is 38°C. The left labia majora has a formation
to 3 cm diameter, with hyperemic surface, extremely painful to the touch, with
symptoms of fluctuation. What is the most likely diagnosis?
A. Acute bartholinitis
B. Vulvar fibroid
C. Vulvar cancer
D. Bartholin gland cyst
E. Hypertrophy of the labia
114. A 28-years-old woman complains of nausea and vomiting about 10
times per day. She has been found to have body weight loss and xerodermia.
The pulse is 100 bpm. Body temperature is 37,2°C. Diuresis is low. USI shows
5-6 weeks of pregnancy. What is the most likely diagnosis?
A. Moderate vomiting of pregnancy
B. I degree preeclampsia
C. Food poisoning
D. Premature abortion
E. Mild vomiting of pregnancy
115. A 40 week pregnant secundipara is 28 years old. Contractions are very
active. Retraction ring is at the level of navel, the uterus is hypertonic, in form
of hourglass. On auscultation the fetal heart sounds are dull, heart rate is
100/min. AP of the parturient woman is 130/80 mm Hg. What is the most
likely diagnosis?
A. Risk of hysterorrhexis
B. Attack of eclampsia
C. Disturbed labour
D. Mazolysis
E. Complete hysterorrhexis
116. After delivery and revision of placenta there was found the defect of
placental lobule. General condition of woman is normal, uterus is firm, there
is moderate bloody discharge. Speculum inspection of birth canal shows
absence of lacerations and raptures. What action is nesessary?
A. Manual exploration of the uterine cavity
B. External massage of uterus
C. Urine drainage, cold on the lower abdomen
D. Introduction of hemostatic medications
E. Introduction of uterine contracting agents
117. A 25 y.o. patient complains of body temperature rise up to 37°C, pain at
the bottom of her abdomen and vaginal discharges. Three days ago, when she
was in her 11th week of pregnancy, she had an artificial abortion. Objectibely:
cervix of uterus is clean, uterus is a little bit enlarged in size, painful.
Appendages cannot be determined. Fornixes are deep, painless. Vaginal
discharges are sanguinopurulent. What is the most probable diagnosis?
A. Postabortion endometritis
B. Postabortion uterus perforation
C. Parametritis
D. Hematometra
E. Pelvic peritonitis
118. A 25 y.o. pregnant woman in her 34th week was taken to the maternity
house in grave condition. She complains of headache, visual impairment,
nausea. Objectively: solid edemata, AP- 170/130 mm Hg. Suddenly there
appeared fibrillary tremor of face muscles, tonic and clonic convulsions,
breathing came to a stop. After 1,5 minute the breathing recovered, there
appeared some bloody spume from her mouth. In urine: protein - 3,5 g/L.
What is the most probable diagnosis?
A. Eclampsia
B. Cerebral hemorrhage
C. Epilepsy
D. Cerebral edema
E. Stomach ulcer
119. A 51-year-old patient complains of having intensive bloody discharges
from vagina for 15 days after delay of menstruation for 2,5 months. In
anamnesis: disorders of menstrual function during a year, at the same time
she felt extreme irritability and had sleep disorders. US examination results:
uterus corresponds with age norms, appendages have no pecularities,
endometrium is 14 mm thick. What is the doctors tactics?
A. Diagnostic curettage of uterine cavity
B. TORCH-infection test
C. Hysterectomy
D. Conservative treatment of bleeding
E. Supravaginal amputation of uterus without appendages
120. An 18 y.o. patient complains of painfulness and swelling of mammary
glands, headaches, irritability, edemata of lower extremities. These symptoms
have been present since the begin of menarche, appear 3-4 days before regular
menstruation. Gynecological examination revealed no pathology. What is the
most probable diagnosis?
A. Premenstrual syndrome
B. Neurasthenia
C. Mastopathy
D. Disease of cardiovascular system
E. Renal disease
121. A 22-year-old female patient complains of dull pain in her right iliac
area that she has been experiencing for a week, morning sickness and
gustatory change. She has a history of menstruation delay for 3 weeks.
Objectively: AP- 80/50 mm Hg, pulse is 78 bpm, body temperature is 37°C.
Bimanual examination reveals that uterus is enlarged, soft, mobile and
painless. Uterine appendages are palpable on the right, there is a dense,
elastic and moderately painful formation 3x4 cm large. What is the most likely
diagnosis?
A. Progressing fallopian pregnancy
B. Right ovarian cyst
C. Acute appendicitis
D. Uterogestation
E. Interrupted fallopian pregnancy
122. A 30 y.o. parturient woman was taken to the maternity house with
complaints of having acute, regular labour pains that last 25-30 seconds every
1,5-2 minutes. Labour activity began 6 hours ago. Uterus is in higher tonus,
head of the fetus is above the opening into the small pelvis. Fetal heartbeat is
136/min. P.V: cervical dilatation is 4 cm, uterine fauces is spasming at a
height of parodynia. Head is level with opening into the small pelvis, it is
being pushed off. What is the most probable diagnosis?
A. Discoordinated labour activity
B. Pathological preliminary period
C. Normal labour activity
D. Primary powerless labour activity
E. Secondary powerless labour activity
123. A primigravida woman appealed to the antenatal clinic on the 22.03.03
with complaints of boring pain in the lower part of abdomen. Anamnesis
registered that her last menstruation was on the 4.01.03. Bimanual
examination revealed that uterine servix is intact, external fauces is closed,
uterus is enlarged up to the 9-th week of pregnancy, movable, painless. What
complication can be suspected?
A. Risk of abortion in the 9-th week of pregnancy
B. Hysteromyoma
C. Vesicular mole
D. Abortion that started in the 9-th week of pregnancy
124. A 25-year-old female patient complains about having amenorrhea for 3
years. She associates it with difficult labour complicated by massive
hemorrhage. She also complains of loss of weight, hair fragility and loss, lack
of appetite and depression. Objective examination reveals no pathological
changes of uterus and its appendages. What is the disease pathogenesis?
A. Hypoproduction of gonadotropin
B. Hyperproduction of estrogens
C. Hypoproduction of progesterone
D. Hyperproduction of prolactin
E. Hyperproduction of androgens
125. A 30-year-old patient consulted a doctor about menstruation absence
for 2 years after labour, loss of hair, body weight loss. The labour was
complicated by a haemorrhage caused by uterus hypotonia. Objectively: the
patient is asthenic, external genitals are hypoplastic, the uterus body is small
and painless. The appendages are not palpaple. What is the most likely
diagnosis?
A. Sheehans syndrome
B. Turners syndrome
C. Galactorrhea-amenorrhea syndrome
D. Exhausted overy syndrome
E. Ovarian amenorrhea
126. A 28-year-old patient has been admitted to the gynecological
department three days after a casual coitus. She complains about pain in her
lower abdomen and during urination, profuse purulent discharges from the
vagina, body temperature rise up to 37,8°C. The patient was diagnosed with
acute bilateral adnexitis. Supplemental examination revealed: the 4th degree
of purity of the vaginal secretion, leukocytes within the whole visual field,
diplococcal bacteria located both intra- and extracellularly. What is the
etiology of acute adnexitis in this patient?
A. Gonorrheal
B. Chlamydial
C. Staphylococcal
D. Trichomonadal
E. Colibacterial
127. A 25-year-old woman came to a maternity welfare clinic and
complained about being unable to conceive within 3 years of regular sexual
life. Examination revealed weight gain, male pattern of hair distribution on
the pubis, excessive pilosis of thighs. Ovaries were dense and enlarged, basal
temperature was monophase. What is the most likely diagnosis?
A. Sclerocystosis of ovaries
B. Adrenogenital syndrome
C. Gonadal dysgenesis
D. Premenstrual syndrome
E. Tubo-ovaritis
128. A 28 year old patient complained about prolongation of intermenstrual
periods up to 2 months, hirsutism. Gynaecological examination revealed that
the ovaries were enlarged, painless, compact, uterus had no pecularities.
Pelvic ultrasound revealed that the ovaries were 4-5 cm in diameter and had
multiple enlarged follicles on periphery. Roentgenography of skull base
showed that sellar region was dilated. What is the most probable diagnosis?
A. Stein-Leventhal syndrome
B. Sheehans syndrome
C. Morgagni-Stewart syndrome
D. Premenstrual syndrome
E. Algodismenorrhea
129. A 32-year-old gravida complains of episodes of unconsciousness,
spontaneous syncopes that are quickly over after a change of body position. A
syncope can be accompanied by quickly elapsing bradycardia. There are no
other complications of gestation. What is the most likely reason for such
condition?
A. Postcava compresseion by the gravid uterus
B. Vegetative-vascular dystonia (cardiac type)
C. Psychosomatic disorders
D. Pressure rise in the veins of extremities
E. Pressure fall in the veins of extremities
130. A woman consulted a therapeutist about fatigability, significant weight
loss, weakness, loss of appetite. She has had amenorrhea for 8 months. A year
ago she born a full-term child. Haemorrhage during labour made up 2 l. She
got blood and blood substitute transfusions. What is the most probable
diagnosis?
A. Sheehans syndrome
B. Shereshevsky-Turners syndrome
C. Vegetovascular dystonia
D. Homological blood syndrome
E. Stein-Leventhal syndrome
131. A 54-year-old female patient consulted a gynaecologist about bloody
discharges from the vagina for 1 month. Last menstruation was 5 years ago.
Gynaecological examination revealed no pathological changes. What is the
tactics of choice?
A. Diagnostic fractional curettage of uterine cavity
B. USI
C. Colposcopy
D. Cytosmear
E. Symptomatic therapy
132. A 28-year-old female patient complains of having haemorrhage from
the genital tracts for 1 month. 6 months ago she had natural delivery and gave
birth to a girl weighing 3100 g. Objectively: the uterus is enlarged to 9-10
weeks, mobile, painless, of heterogenous consistency. Examination reveals
vaginal cyanosis, anaemia and body temperature rise up to 37,8°C. There is a
significant increase in hCG concentration in the urine. What is your
provisional diagnosis?
A. Uterine chorionepithelioma
B. Endometritis
C. Uterine fibromyoma
D. Pregnancy
E. Hydatidiform mole
133. An ambulance delivered a 21-year-old woman to the gynaecological
department with complaints of colicky abdominal pain and bloody discharges
from the genital tracts. Bimanual examination revealed that uterus was soft,
enlarged to the size of 6 weeks of gestation, a gestational sac was palpated in
the cervical canal. Uterine appendages werent palpable. Fornices are free,
deep and painless. Discharges from the genital tracts are bloody and profuse.
What is the most likely diagnosis?
A. Abortion in progress
B. Interrupted fallopian pregnancy
C. Threat of abortion
D. Cervical pregnancy
E. Incipient abortion
134. A 33 y.o. woman survived twooperations on account of extrauterine
pregnancy, both uterine tubes were removed. She consulted a doctor with a
question about possibility of having a child. What can be advised in this case?
A. Extracorporal fertilization
B. Induction of ovulation
C. Substitutional maternity
D. Insemination with her husbands semen
E. Artifical fertilization with donors semen
135. On the fifth day after a casual sexual contact a 25-year-old female
patient consulted a doctor about purulent discharges from the genital tracts
and itch. Vaginal examination showed that vaginal part of uterine cervix was
hyperemic and edematic. There was an erosive area around the external
orifice of uterus. There were mucopurulent profuse discharges from the
cervical canal, uterine body and appendages exhibited no changes.
Bacterioscopic examination revealed bean-shaped diplococci that became red
after Grams staining. What is the most likely diagnosis?
A. Acute gonorrheal endocervicitis
B. Candidal vulvovaginitis
C. Bacterial vaginism
D. Clamydial endocervicitis
E. Trichomonal colpitis
136. A 26 year old woman complains about edemata, swelling and
painfulness of mammary glands, headache, tearfulness, irritability. These
signs turn up 5 days before menstruation and disappear after its start. What
clinical syndrome is it?
A. Premenstrual syndrome
B. Stein-Leventhal syndrome
C. Adrenogenital syndrome
D. Postcastration syndrome
E. Climacteric syndrome
137. A 49-year-old woman complains about headache, head and neck going
hot, increased perspiration, palpitation, arterial pressure rise up to 170/100
mm Hg, irritability, insomnia, tearfulness, memory impairment, rare and
scarce menses, body weight increase by 5 kg over the last half a year. What is
the most likely diagnosis?
A. Climacteric syndrome
B. Arterial hypertension
C. Postcastration syndrome
D. Premenstrual syndrome
E. Vegetative-vascular dystonia
138. A 30-year-old female patient has been delivered to the gynaecological
department with complaints of acute pain in the lower abdomen and body
temperature 38,8°C. In history: sexual life out of wedlock and two artificial
abortions. Gynaecological examination reveals no changes of uterine. The
appendages are enlarged and painful on both sides. Vaginal discharges are
purulent and profuse. What study is required to confirm a diagnosis?
A. Bacteriological and bacterioscopic analysis
B. Colposcopy
C. Laparoscopy
D. Hysteroscopy
E. Curettage of uterine cavity
139. A parturient woman is 27 year old, it was her second labour, delivery
was at term, normal course. On the 3rd day of postpartum period body
temperature is 36,8°C, Ps - 72/min, AP - 120/80 mm Hg. Mammary glands
are moderately swollen, nipples are clean. Abdomen is soft and painless.
Fundus of uterus is 3 fingers below the umbilicus. Lochia are bloody,
moderate. What is the most probable diagnosis?
A. Physiological course of postpartum period
B. Lactostasis
C. Postpartum metroendometritis
D. Subinvolution of uterus
E. Remnants of placental tissue after labour
140. A parturient woman is 25 years old, it is her second day of postpartum
period. It was her first full-term uncomplicated labour. The lochia should be:
A. Bloody
B. Mucous
C. Sanguino-serous
D. Purulent
E. Serous
141. A 32-year-old patient consulted a doctor about being inable to get
pregnant for 5-6 years. 5 ago the primipregnancy ended in artificial abortion.
After the vaginal examination and USI the patient was diagnosed with
endometrioid cyst of the right ovary. What is the optimal treatment method?
A. Surgical laparoscopy
B. Conservative therapy with estrogen-gestagenic drugs
C. Sanatorium-and-spa treatment
D. Hormonal therapy with androgenic hormones
E. Anti-inflammatory therapy
142. A woman is 34 years old, it is her tenth labor at full term. It is known
from the anamnesis that the labor started 11 hours ago, labor was active,
painful contractions started after discharge of waters and became continuous.
Suddenly the parturient got knife-like pain in the lower abdomen and labor
activity stopped. Examination revealed positive symptoms of peritoneum
irritation, ill-defined uterus outlines. Fetus was easily palpable, movable. Fetal
heartbeats wasnt auscultable. What is the most probable diagnosis?
A. Rupture of uterus
B. II labor period
C. Discoordinated labor activity
D. Uterine inertia
E. Risk of uterus rupture
143. Examination of placenta revealed a defect. An obstetrician performed
manual investigation of uterine cavity, uterine massage. Prophylaxis of
endometritis in the postpartum period should involve following actions:
A. Antibacterial therapy
B. Contracting agents
C. Intrauterine instillation of dioxine
D. Instrumental revision of uterine cavity
E. Haemostatic therapy
144. A 10 week pregnant woman was admitted to a hospital for recurrent
pain in the lower abdomen, bloody discharges from the genital tracts. The
problems turned up after ARVI. The woman was registered for antenatal care.
Speculum examination revealed cyanosis of vaginal mucosa, clean cervix,
open cervical canal discharging blood and blood clots; the lower pole of the
gestational sac was visible. What tactics should be chosen?
A. Curettage of the uterus
B. Expectant management, surveillance
C. Pregnancy maintenance therapy
D. Hysterectomy
E. Antiviral therapy
145. A 30 year old patient complains about inability to become pregnant
over 3 years of married life. The patient is of supernutrition type, she has hair
along the median abdominal line, on the internal thigh surface and in the
peripapillary area. Menses started at the age of 16, they are infrequent and
non-profuse. US revealed that the uterus was of normal size, ovaries were
4?5?5 cm large and had a lot of cystic inclusions. What is the most probable
diagnosis?
A. Polycystic ovaries
B. Ovarian cystoma
C. Menstrual irregularity
D. Bilateral ovarian tumours
E. Chronic oophoritis
146. A 29-year-old patient complains of absent menstruation for a year,
milk discharge from the nipples when pressed, loss of lateral visual fields. X-
ray shows an expansion of the sella turcica. What is the most likely cause of
this condition?
A. Pituitary tumour
B. Pregnancy
C. Functional disorder of the hypothalamic-pituitary-ovarian system
D. Mammary tumour
E. Ovarian tumor
147. A patient with fibromyoma of uterus sized up to 8-9 weeks of
pregnancy consulted a gynaecologist about acute pain in the lower abdomen.
Examination revealed pronounced positive symptoms of peritoneal irritation,
high leukocytosis. Vaginal examination revealed that the uterus was enlarged
corresponding to 9 weeks of pregnancy due to the fibromatous nodes, one of
which was mobile and extremely painful. Appendages were not palpable.
There were moderate mucous discharges. What is the optimal treatment
tactics?
A. Urgent surgery (laparotomy)
B. Fractional diagnostic curettage of the uterine cavity
C. Surveillance and spasmolytic therapy
D. Surgical laparoscopy
E. Surveillance and antibacterial therapy
148. A pregnant woman was delivered to the gynecological unit with
complaints of pain in the lower abdomen and insignificant bloody discharges
from the genital tracts for 3 hours. Last menstruation was 3 months ago.
Vaginal examination showed that body of womb was in the 10th week of
gestation, a fingertip could be inserted into the external orifice of uterus,
bloody discharges were insignificant. USI showed small vesicles in the uterine
cavity. What is the most likely diagnosis?
A. Grape mole
B. Abortion in progress
C. Threat of spontaneous abortion
D. Incomplete abortion
E. Incipient abortion
149. A 49-year-old patient undergoes regular medical check-up for uterine
fibromyoma. Within the last year the uterus has enlarged up to 20 weeks of
gestation. What is the rational way of treatment?
A. Surgical treatment
B. Treatment with prostaglandin inhibitors
C. Further surveillance
D. Hormonal therapy
E. Embolization of uterine arteries
150. A female patient complains of being unable to get pregnant for 5 years.
A complete clinical examination brought the following results: hormonal
function is not impaired, urogenital infection hasnt been found, on
hysterosalpingography both tubes were filled with the contrast medium up to
the isthmic segment, abdominal contrast was not visualized. The patients
husband is healthy. What tactics will be most effective?
A. In-vitro fertilization
B. Laparoscopic tubal plasty
C. ICSI within in-vitrofertilization program
D. Insemination with husbands sperm
E. Hydrotubation
151. A 19-year-old primiparous woman with a body weight of 54,5 kg gave
birth at 38 weeks gestation to a full-term live girl after a normal vaginal
delivery. The girls weight was 2180,0 g, body length - 48 cm. It is known from
history that the woman has been a smoker for 8 years, and kept smoking
during pregnancy. Pregnancy was complicated by moderate vomiting of
pregnancy from 9 to 12 weeks pregnant, edemata of pregnancy from 32 to 38
weeks. What is the most likely cause of low birth weight?
A. Fetoplacental insufficiency
B. Third trimester preeclampsia
C. Womans age
D. Low weight of the woman
E. First trimester preeclampsia
152. A primigravida is 22 years old. She has Rh(-), her husband has Rh(+).
Antibodies to Rh werent found at 32 weeks of pregnancy. Redetermination of
antibodies to Rh didnt reveal them at 35 weeks of pregnancy as well. How
often should the antibodies be determined hereafter?
A. Once a week
B. Once in three weeks
C. Once in two weeks
D. Montly
E. There is no need in further checks
153. A maternity house has admitted a primagravida complaining of
irregular, intense labour pains that have been lasting for 36 hours. The woman
is tired, failed to fall asleep at night. The fetus is in longitudinal lie, with
cephalic presentation. The fetus heartbeat is clear and rhythmic, 145/min.
Vaginal examination revealed that the uterine cervix was up to 3 cm long,
dense, with retroflexion; the external orifice was closed; the discharges were of
mucous nature. What is the most likely diagnosis?
A. Pathological preliminary period
B. Physiological preliminary period
C. Secondary uterine inertia
D. Uterine cervix dystocia
E. Primary uterine inertia
154. A 14-year-old girl complains of pain in vaginal area and lower abdomen
that last for 3-4 days and have been observed for 3 months about the same
time. Each time pain is getting worse. Objectively: mammary glands are
developed, hairiness corresponds to the age. The virginal membrane is intact,
cyanotic and protruded. She has never had menstruation. She has been
diagnosed with primary amenorrhea. What is the reason of amenorrhea?
A. Hymen atresia
B. Sexual development delay
C. Babinski-Frohlich syndrome
D. Turners syndrome
E. Pregnancy
155. A multigravida with Rh-isosensitization was found to have a decrease
in anti-Rh titer from 1:32 to 1:8 at 33-34 weeks of gestation. Ultrasound
revealed double contour of head, ebnlargement of fetal liver, placental
thickness of 50 mm. The patient has indication for:
A. Premature delivery
B. Plasmapheresis
C. Administration of anti-Rh gamma globulin
D. Repeated (after 2 weeks) USI
E. Course of desensitizing therapy
156. A 13-year-old girl was admitted to the gynecological department with
heavy bleeding, which appeared after a long delay of menstruation. Shortly
before, the girl suffered a serious psychotrauma. Her menarche occurred at
the age of 11, she has a 30-day cycle with 5 to 6 days of moderate, painless
bleeding. The patient is somatically healthy, of normosthenic constitution
with height of 160 cm, weight of 42 kg. The patient is pale. Rectoabdominal
examination revealed that the uterus was of normal size and consistency,
anteflexio-versio, the appendages were not changed. What is the most likely
diagnosis?
A. Juvenile bleeding
B. Hysteromyoma
C. Amenorrhea
D. Girl is healthy
E. Ovarian cyst
157. A 38-year-old female patient complains about hot flashes and feeling of
intense heat arising up to 5 times a day, headaches in the occipital region
along with high blood pressure, palpitations, dizziness, fatigue, irritability,
memory impairment. 6 months ago the patient underwent extirpation of the
uterus with its appendages. What is the most likely diagnosis?
A. Post-castration syndrome
B. Premenstrual syndrome
C. Secondary psychogenic amenorrhea
D. Physiological premenopause
E. Early pathological menopause
158. A 27-year-old patient complains of irritability, tearfulness, depression,
and sometimes aggressiveness, headache, nausea, vomiting, swelling of the
mammary glands. The mentioned problems arise 5-6 days before
menstruation and gradually progress until menstruation, 3 days after it the
problems disappear. What is the most likely diagnosis?
A. Premenstrual syndrome
B. Secondary psychogenic amenorrhea
C. Premature pathological climacterium
D. Preclimacterium syndrome
E. Algomenorrhea
159. A 23-year-old primigravida at 39 weeks gestation has been admitted to
the maternity ward with irregular contractions. The intensity of uterine
contractions is not changing, the intervals between them stay long. Bimanual
examination reveals that the cervix is centered, soft, up to 1,5 cm long. There
is no cervical dilatation. What diagnosis should be made?
A. Pregnancy I, 39 weeks, preliminary period
B. Pregnancy I, 39 weeks, labor I, period 1, the active phase
C. Pregnancy I, 39 weeks, pathological preliminary period
D. Pregnancy I, 39 weeks, birth I, 1 period, the acceleration phase
E. Pregnancy I, 39 weeks, labor I, 1 period, the latent phase
160. 20 minutes after a normal delivery at 39 weeks a puerpera had a single
temperature rise up to 38°C. Objectively: the uterus is dense, located between
the navel and the pubis, painless. Lochia are bloody, of small amount. Breasts
are moderately soft and painless. What is the optimal tactics?
A. Further follow-up
B. Appointment antipyretic
C. Expression of breast
D. Manual examination of the uterine cavity
E. Antibiotic therapy
161. On the 10th day postpartum a puerperant woman complains of pain
and heaviness in the left breast. Body temperature is 38,8°C, Ps - 94 bpm. The
left breast is edematic, the supero-external quadrant of skin is hyperemic.
Fluctuation symptom is absent. The nipples discharge drops of milk when
pressed. What is a doctors further tactics?
A. Antibiotic therapy, immobilization and expression of breast milk
B. Physiotherapy
C. Opening of the abscess and drainage of the breast
D. Compress to both breasts
E. Inhibition of lactation
162. On the 10th day postpartum a puerperant woman complains of pain
and heaviness in the left mammary gland. Body temperature is 38,8°C, Ps- 94
bpm. The left mammary gland is edematic, the supero-external quadrant of
skin is hyperemic. Fluctuation symptom is absent. The nipples discharge
drops of milk when pressed. What is a doctors further tactics?
A. Antibiotic therapy, immobilization and expression of breast milk
B. Physiotherapy
C. Opening of the abscess and drainage of the mammary gland
D. Compress to both mammary glands
E. Inhibition of lactation
163. A 30-year-old female patient complains of milk discharge from the
mammary glands, 5-month absence of menstruation. She had one
physiological labour four years ago. Objectively: mammary glands are
normally developed. Bimanual examination reveals that the uterus is
decreased in size, the ovaries are of normal size. MRI-scan shows no cerebral
pathologies. Concentration of thyroid-stimulating hormone is normal. The
serum prolactin level is increased. What is the most likely diagnosis?
A. Hyperprolactinemia
B. Hypothyroidism
C. Pituitary adenoma
D. Sheehan syndrome
E. Polycystic ovary syndrome
164. During self-examination a 22-year-old patient revealed a mammary
tumour. Palpation revealed a firm, painless, mobile formation up to 2 cm,
peripheral lymph nodes were not changed. USI results: in the superior
external quadrant of the right mammary gland there was a big formation of
increased echogenicity, sized 18x17 mm. The patient was provisionally
diagnosed with fibroadenoma. What is a doctors further tactics?
A. Surgical removal of the tumour prior to pregnancy
B. Dynamic follow-up
C. Radical mastectomy
D. Nonsteroid anti-inflammatory drugs, oral contraceptives
E. Surgical treatment after pregnancy
165. A 25-year-old female has a self-detected tumor in the upper outer
quadrant of her right breast. On palpation there is a painless, firm, mobile
lump up to 2 cm in diameter, peripheral lymph nodes are not changed. In the
upper outer quadrant of the right breast ultrasound revealed a massive
neoplasm with increased echogenicity sized 21x18 mm. What is the most likely
diagnosis?
A. Fibroadenoma
B. Diffuse mastopathy
C. Mastitis
D. Mammary cancer
E. Lactocele
166. A 49-year-old female patient complains of itching, burning in the
external genitals, frequent urination. The symptoms have been present for the
last 7 months. The patient has irregular menstruation, once every 3-4 months.
Over the last two years she has had hot flashes, sweating, sleep disturbance.
Examination revealed no pathological changes of the internal reproductive
organs. Complete blood count and urinalysis showed no pathological changes.
Vaginal smear contained 20-25 leukocytes per HPF, mixed flora. What is the
most likely diagnosis?
A. Menopausal syndrome
B. Trichomonas colpitis
C. Cystitis
D. Vulvitis
E. Bacterial vaginosis
167. 2 weeks after labour a parturient woman developed breast pain being
observed for 3 days. Examination revealed body temperature at the rate of
39°C, chills, weakness, hyperaemia, enlargement, pain and deformity of the
mammary gland. On palpation the infiltrate was found to have an area of
softening and fluctuation. What is the most likely diagnosis?
A. Infiltrative-purulent mastitis
B. Phlegmonous mastitis
C. Serous mastitis
D. Mastopathy
E. Lactostasis
168. Preventive examination of a 50-year-old woman revealed a dense
tumour of the right mammary gland up to 5 cm in diameter without distinct
outlines. The skin over the tumour looked like lemon peel. Palpation revealed
a lymph node in the axillary region. What is the most likely diagnosis?
A. Breast cancer
B. Mastitis
C. Breast lipoma
D. Lactocele
E. Diffuse mastopathy
169. A 20-year-old female consulted a gynecologist about not having
menstrual period for 7 months. History abstracts: early childhood infections
and frequent tonsillitis, menarche since 13 years, regular monthly menstrual
cycle of 28 days, painless menstruation lasts 5-6 days. 7 months ago the
patient had an emotional stress. Gynecological examination revealed no
alterations in the uterus. What is the most likely diagnosis?
A. Secondary amenorrhea
B. Spanomenorrhea
C. Cryptomenorrhea
D. Primary amenorrhea
E. Algomenorrhea
170. A 48-year-old female has been admitted to the gynecology department
for pain in the lower right abdomen and low back pain, constipations.
Bimanual examination findings: the uterus is immobile, the size of a 10-week
pregnancy, has uneven surface. Aspirate from the uterine cavity contains
atypical cells. What diagnosis can be made?
A. Hysterocarcinoma
B. Colon cancer
C. Chorionepithelioma
D. Cervical cancer
E. Metrofibroma

More Related Content

What's hot

Krok 2 - 2014 Question Paper (General Medicine)
Krok 2 - 2014 Question Paper (General Medicine)Krok 2 - 2014 Question Paper (General Medicine)
Krok 2 - 2014 Question Paper (General Medicine)Eneutron
 
Krok 2 - 2013 (Surgery)
Krok 2 - 2013 (Surgery)Krok 2 - 2013 (Surgery)
Krok 2 - 2013 (Surgery)Eneutron
 
Complete perineal tear
Complete perineal tearComplete perineal tear
Complete perineal tearmagdy abdel
 
PELVIC INFLAMMATORY DISEASE (PID)
PELVIC INFLAMMATORY DISEASE (PID)PELVIC INFLAMMATORY DISEASE (PID)
PELVIC INFLAMMATORY DISEASE (PID)Mohammed Musa
 
Urology 4 hydronephrosis
Urology 4 hydronephrosisUrology 4 hydronephrosis
Urology 4 hydronephrosissurgerymgmcri
 
Krok 2 - 2013 Question Paper (General Medicine)
Krok 2 - 2013 Question Paper (General Medicine)Krok 2 - 2013 Question Paper (General Medicine)
Krok 2 - 2013 Question Paper (General Medicine)Eneutron
 
Premature ovarian failure
Premature ovarian failurePremature ovarian failure
Premature ovarian failureShambhu N
 
MEDICAL MANAGEMENT OF ABNORMAL UTERINE BLEEDING BY DR SHASHWAT JANI
MEDICAL MANAGEMENT OF ABNORMAL UTERINE BLEEDING  BY DR SHASHWAT JANIMEDICAL MANAGEMENT OF ABNORMAL UTERINE BLEEDING  BY DR SHASHWAT JANI
MEDICAL MANAGEMENT OF ABNORMAL UTERINE BLEEDING BY DR SHASHWAT JANIDR SHASHWAT JANI
 
Uterine fibroids
Uterine fibroidsUterine fibroids
Uterine fibroidsdrmcbansal
 
Pre-Eclampsia & Eclampsia
Pre-Eclampsia & EclampsiaPre-Eclampsia & Eclampsia
Pre-Eclampsia & Eclampsialimgengyan
 
20.Pelvic Inflammatory Disease
20.Pelvic Inflammatory Disease20.Pelvic Inflammatory Disease
20.Pelvic Inflammatory DiseaseDeep Deep
 
Septic abortion (3)
Septic abortion (3)Septic abortion (3)
Septic abortion (3)Pratyush1693
 
Krok 2 - 2014 (Therapy)
Krok 2 - 2014 (Therapy)Krok 2 - 2014 (Therapy)
Krok 2 - 2014 (Therapy)Eneutron
 

What's hot (20)

Krok 2 - 2014 Question Paper (General Medicine)
Krok 2 - 2014 Question Paper (General Medicine)Krok 2 - 2014 Question Paper (General Medicine)
Krok 2 - 2014 Question Paper (General Medicine)
 
Krok 2 - 2013 (Surgery)
Krok 2 - 2013 (Surgery)Krok 2 - 2013 (Surgery)
Krok 2 - 2013 (Surgery)
 
Complete perineal tear
Complete perineal tearComplete perineal tear
Complete perineal tear
 
PELVIC INFLAMMATORY DISEASE (PID)
PELVIC INFLAMMATORY DISEASE (PID)PELVIC INFLAMMATORY DISEASE (PID)
PELVIC INFLAMMATORY DISEASE (PID)
 
Amenorrhea
AmenorrheaAmenorrhea
Amenorrhea
 
adenomyosis
adenomyosisadenomyosis
adenomyosis
 
Urology 4 hydronephrosis
Urology 4 hydronephrosisUrology 4 hydronephrosis
Urology 4 hydronephrosis
 
Endometrial Carcinoma
Endometrial CarcinomaEndometrial Carcinoma
Endometrial Carcinoma
 
Postpartum Infections
Postpartum InfectionsPostpartum Infections
Postpartum Infections
 
Krok 2 - 2013 Question Paper (General Medicine)
Krok 2 - 2013 Question Paper (General Medicine)Krok 2 - 2013 Question Paper (General Medicine)
Krok 2 - 2013 Question Paper (General Medicine)
 
Premature ovarian failure
Premature ovarian failurePremature ovarian failure
Premature ovarian failure
 
MEDICAL MANAGEMENT OF ABNORMAL UTERINE BLEEDING BY DR SHASHWAT JANI
MEDICAL MANAGEMENT OF ABNORMAL UTERINE BLEEDING  BY DR SHASHWAT JANIMEDICAL MANAGEMENT OF ABNORMAL UTERINE BLEEDING  BY DR SHASHWAT JANI
MEDICAL MANAGEMENT OF ABNORMAL UTERINE BLEEDING BY DR SHASHWAT JANI
 
Uterine fibroids
Uterine fibroidsUterine fibroids
Uterine fibroids
 
Pre-Eclampsia & Eclampsia
Pre-Eclampsia & EclampsiaPre-Eclampsia & Eclampsia
Pre-Eclampsia & Eclampsia
 
20.Pelvic Inflammatory Disease
20.Pelvic Inflammatory Disease20.Pelvic Inflammatory Disease
20.Pelvic Inflammatory Disease
 
Endometriosis
EndometriosisEndometriosis
Endometriosis
 
Septic abortion (3)
Septic abortion (3)Septic abortion (3)
Septic abortion (3)
 
Krok 2 - 2014 (Therapy)
Krok 2 - 2014 (Therapy)Krok 2 - 2014 (Therapy)
Krok 2 - 2014 (Therapy)
 
Pelvic inflammatory disease (pid)
Pelvic inflammatory disease (pid)Pelvic inflammatory disease (pid)
Pelvic inflammatory disease (pid)
 
PROM
PROMPROM
PROM
 

Similar to Gynecology Exam Questions

акушерство і гінекологія
акушерство і гінекологіяакушерство і гінекологія
акушерство і гінекологіяAmmar Sattar
 
Krok II collection by masar muslim salih
Krok II collection by masar muslim salihKrok II collection by masar muslim salih
Krok II collection by masar muslim salihMasar Muslim
 
Krok 2 - 2007 Question Paper (General Medicine)
Krok 2 - 2007 Question Paper (General Medicine)Krok 2 - 2007 Question Paper (General Medicine)
Krok 2 - 2007 Question Paper (General Medicine)Eneutron
 
Krok 2 - 2008 Question Paper (General Medicine)
Krok 2 - 2008 Question Paper (General Medicine)Krok 2 - 2008 Question Paper (General Medicine)
Krok 2 - 2008 Question Paper (General Medicine)Eneutron
 
Gynac 170mcq krok 2
Gynac 170mcq krok 2Gynac 170mcq krok 2
Gynac 170mcq krok 2Raj Twix
 
OBG%20FMGE%20LRR%20Part%202.pdf
OBG%20FMGE%20LRR%20Part%202.pdfOBG%20FMGE%20LRR%20Part%202.pdf
OBG%20FMGE%20LRR%20Part%202.pdfpratappankaj2017
 
recall Jan 2023.pptx
recall Jan 2023.pptxrecall Jan 2023.pptx
recall Jan 2023.pptxAishaNajeeb1
 
information_MSQ_KROK_2_Medicine_2007_2021_OBSTETRICS_GYNECOLOGY.pptx
information_MSQ_KROK_2_Medicine_2007_2021_OBSTETRICS_GYNECOLOGY.pptxinformation_MSQ_KROK_2_Medicine_2007_2021_OBSTETRICS_GYNECOLOGY.pptx
information_MSQ_KROK_2_Medicine_2007_2021_OBSTETRICS_GYNECOLOGY.pptxRitikAgarsen1
 
Gyn obs final exam, 2014
Gyn obs final exam, 2014Gyn obs final exam, 2014
Gyn obs final exam, 2014Tadiwos Gintamo
 
Placenta Accreta Spectrum.pptx
Placenta Accreta Spectrum.pptxPlacenta Accreta Spectrum.pptx
Placenta Accreta Spectrum.pptxFawad Mueen Arbi
 
Gync additional krok 2
Gync additional krok 2Gync additional krok 2
Gync additional krok 2Raj Twix
 
Krok 2 - 2009 Question Paper (General Medicine)
Krok 2 - 2009 Question Paper (General Medicine)Krok 2 - 2009 Question Paper (General Medicine)
Krok 2 - 2009 Question Paper (General Medicine)Eneutron
 
Spontaneous rupture of endometriotic cyst in 3rd trimester of pregnancy
Spontaneous rupture of endometriotic cyst in 3rd trimester of pregnancySpontaneous rupture of endometriotic cyst in 3rd trimester of pregnancy
Spontaneous rupture of endometriotic cyst in 3rd trimester of pregnancyApollo Hospitals
 
Krok 2 - 2014 (Surgery)
Krok 2 - 2014 (Surgery)Krok 2 - 2014 (Surgery)
Krok 2 - 2014 (Surgery)Eneutron
 
Krok 2 - 2010 Question Paper (General Medicine)
Krok 2 - 2010 Question Paper (General Medicine)Krok 2 - 2010 Question Paper (General Medicine)
Krok 2 - 2010 Question Paper (General Medicine)Eneutron
 
Neb step 1 formative assessment-day-8
Neb step 1 formative assessment-day-8Neb step 1 formative assessment-day-8
Neb step 1 formative assessment-day-8DrSaeed Shafi
 

Similar to Gynecology Exam Questions (20)

акушерство і гінекологія
акушерство і гінекологіяакушерство і гінекологія
акушерство і гінекологія
 
Krok II collection by masar muslim salih
Krok II collection by masar muslim salihKrok II collection by masar muslim salih
Krok II collection by masar muslim salih
 
Krok 2 - 2007 Question Paper (General Medicine)
Krok 2 - 2007 Question Paper (General Medicine)Krok 2 - 2007 Question Paper (General Medicine)
Krok 2 - 2007 Question Paper (General Medicine)
 
Krok 2 - 2008 Question Paper (General Medicine)
Krok 2 - 2008 Question Paper (General Medicine)Krok 2 - 2008 Question Paper (General Medicine)
Krok 2 - 2008 Question Paper (General Medicine)
 
Gynac 170mcq krok 2
Gynac 170mcq krok 2Gynac 170mcq krok 2
Gynac 170mcq krok 2
 
OBG%20FMGE%20LRR%20Part%202.pdf
OBG%20FMGE%20LRR%20Part%202.pdfOBG%20FMGE%20LRR%20Part%202.pdf
OBG%20FMGE%20LRR%20Part%202.pdf
 
recall Jan 2023.pptx
recall Jan 2023.pptxrecall Jan 2023.pptx
recall Jan 2023.pptx
 
Obstetric hemorrhage cases and MCQ for undergraduate
Obstetric hemorrhage cases and MCQ for undergraduateObstetric hemorrhage cases and MCQ for undergraduate
Obstetric hemorrhage cases and MCQ for undergraduate
 
information_MSQ_KROK_2_Medicine_2007_2021_OBSTETRICS_GYNECOLOGY.pptx
information_MSQ_KROK_2_Medicine_2007_2021_OBSTETRICS_GYNECOLOGY.pptxinformation_MSQ_KROK_2_Medicine_2007_2021_OBSTETRICS_GYNECOLOGY.pptx
information_MSQ_KROK_2_Medicine_2007_2021_OBSTETRICS_GYNECOLOGY.pptx
 
Gyn obs final exam, 2014
Gyn obs final exam, 2014Gyn obs final exam, 2014
Gyn obs final exam, 2014
 
Placenta Accreta Spectrum.pptx
Placenta Accreta Spectrum.pptxPlacenta Accreta Spectrum.pptx
Placenta Accreta Spectrum.pptx
 
Gync additional krok 2
Gync additional krok 2Gync additional krok 2
Gync additional krok 2
 
Krok 2 - 2009 Question Paper (General Medicine)
Krok 2 - 2009 Question Paper (General Medicine)Krok 2 - 2009 Question Paper (General Medicine)
Krok 2 - 2009 Question Paper (General Medicine)
 
Spontaneous rupture of endometriotic cyst in 3rd trimester of pregnancy
Spontaneous rupture of endometriotic cyst in 3rd trimester of pregnancySpontaneous rupture of endometriotic cyst in 3rd trimester of pregnancy
Spontaneous rupture of endometriotic cyst in 3rd trimester of pregnancy
 
Amniocenesis
AmniocenesisAmniocenesis
Amniocenesis
 
Krok 2 - 2014 (Surgery)
Krok 2 - 2014 (Surgery)Krok 2 - 2014 (Surgery)
Krok 2 - 2014 (Surgery)
 
Krok 2 - 2010 Question Paper (General Medicine)
Krok 2 - 2010 Question Paper (General Medicine)Krok 2 - 2010 Question Paper (General Medicine)
Krok 2 - 2010 Question Paper (General Medicine)
 
Neb step 1 formative assessment-day-8
Neb step 1 formative assessment-day-8Neb step 1 formative assessment-day-8
Neb step 1 formative assessment-day-8
 
Placenta Accreta
Placenta AccretaPlacenta Accreta
Placenta Accreta
 
Aneurysm-like Invasion into the Bladder of Pernicious Placenta Previa with Pl...
Aneurysm-like Invasion into the Bladder of Pernicious Placenta Previa with Pl...Aneurysm-like Invasion into the Bladder of Pernicious Placenta Previa with Pl...
Aneurysm-like Invasion into the Bladder of Pernicious Placenta Previa with Pl...
 

More from Eneutron

PGCET Textile 2018 question paper
PGCET Textile 2018 question paperPGCET Textile 2018 question paper
PGCET Textile 2018 question paperEneutron
 
PGCET Polymer science 2018 question paper
PGCET Polymer science 2018 question paperPGCET Polymer science 2018 question paper
PGCET Polymer science 2018 question paperEneutron
 
PGCET Mechanical 2018 question paper
PGCET Mechanical 2018 question paperPGCET Mechanical 2018 question paper
PGCET Mechanical 2018 question paperEneutron
 
PGCET Environmental 2018 question paper
PGCET Environmental 2018 question paperPGCET Environmental 2018 question paper
PGCET Environmental 2018 question paperEneutron
 
PGCET Electrical sciences 2018 question paper
PGCET Electrical sciences 2018 question paperPGCET Electrical sciences 2018 question paper
PGCET Electrical sciences 2018 question paperEneutron
 
PGCET Computer science 2018 question paper
PGCET Computer science 2018 question paperPGCET Computer science 2018 question paper
PGCET Computer science 2018 question paperEneutron
 
PGCET Civil 2018 question paper
PGCET Civil 2018 question paperPGCET Civil 2018 question paper
PGCET Civil 2018 question paperEneutron
 
PGCET Chemical 2018 question paper
PGCET Chemical 2018 question paperPGCET Chemical 2018 question paper
PGCET Chemical 2018 question paperEneutron
 
PGCET Biotechnology 2018 question paper
PGCET Biotechnology 2018 question paperPGCET Biotechnology 2018 question paper
PGCET Biotechnology 2018 question paperEneutron
 
Pgcet Architecture 2018 question paper
Pgcet Architecture 2018 question paperPgcet Architecture 2018 question paper
Pgcet Architecture 2018 question paperEneutron
 
Pgcet Architecture 2017 question paper
Pgcet Architecture 2017 question paperPgcet Architecture 2017 question paper
Pgcet Architecture 2017 question paperEneutron
 
PGCET MBA 2018 question paper
PGCET MBA 2018 question paperPGCET MBA 2018 question paper
PGCET MBA 2018 question paperEneutron
 
Civil Service 2019 Prelims Previous Question Paper - 2
Civil Service 2019 Prelims Previous Question Paper - 2Civil Service 2019 Prelims Previous Question Paper - 2
Civil Service 2019 Prelims Previous Question Paper - 2Eneutron
 
Civil Service 2019 Prelims Previous Question Paper - 1
Civil Service 2019 Prelims Previous Question Paper - 1Civil Service 2019 Prelims Previous Question Paper - 1
Civil Service 2019 Prelims Previous Question Paper - 1Eneutron
 
Civil Service 2018 Prelims Previous Question Paper - 2
Civil Service 2018 Prelims Previous Question Paper - 2Civil Service 2018 Prelims Previous Question Paper - 2
Civil Service 2018 Prelims Previous Question Paper - 2Eneutron
 
Civil Service 2018 Prelims Previous Question Paper - 1
Civil Service 2018 Prelims Previous Question Paper - 1Civil Service 2018 Prelims Previous Question Paper - 1
Civil Service 2018 Prelims Previous Question Paper - 1Eneutron
 
Civil Service 2017 Prelims Previous Question Paper - 2
Civil Service 2017 Prelims Previous Question Paper - 2Civil Service 2017 Prelims Previous Question Paper - 2
Civil Service 2017 Prelims Previous Question Paper - 2Eneutron
 
Civil Service 2017 Prelims Previous Question Paper - 1
Civil Service 2017 Prelims Previous Question Paper - 1Civil Service 2017 Prelims Previous Question Paper - 1
Civil Service 2017 Prelims Previous Question Paper - 1Eneutron
 
SNAP 2013 Answer Key
SNAP 2013 Answer KeySNAP 2013 Answer Key
SNAP 2013 Answer KeyEneutron
 
SNAP 2014 Answer Key
SNAP 2014 Answer KeySNAP 2014 Answer Key
SNAP 2014 Answer KeyEneutron
 

More from Eneutron (20)

PGCET Textile 2018 question paper
PGCET Textile 2018 question paperPGCET Textile 2018 question paper
PGCET Textile 2018 question paper
 
PGCET Polymer science 2018 question paper
PGCET Polymer science 2018 question paperPGCET Polymer science 2018 question paper
PGCET Polymer science 2018 question paper
 
PGCET Mechanical 2018 question paper
PGCET Mechanical 2018 question paperPGCET Mechanical 2018 question paper
PGCET Mechanical 2018 question paper
 
PGCET Environmental 2018 question paper
PGCET Environmental 2018 question paperPGCET Environmental 2018 question paper
PGCET Environmental 2018 question paper
 
PGCET Electrical sciences 2018 question paper
PGCET Electrical sciences 2018 question paperPGCET Electrical sciences 2018 question paper
PGCET Electrical sciences 2018 question paper
 
PGCET Computer science 2018 question paper
PGCET Computer science 2018 question paperPGCET Computer science 2018 question paper
PGCET Computer science 2018 question paper
 
PGCET Civil 2018 question paper
PGCET Civil 2018 question paperPGCET Civil 2018 question paper
PGCET Civil 2018 question paper
 
PGCET Chemical 2018 question paper
PGCET Chemical 2018 question paperPGCET Chemical 2018 question paper
PGCET Chemical 2018 question paper
 
PGCET Biotechnology 2018 question paper
PGCET Biotechnology 2018 question paperPGCET Biotechnology 2018 question paper
PGCET Biotechnology 2018 question paper
 
Pgcet Architecture 2018 question paper
Pgcet Architecture 2018 question paperPgcet Architecture 2018 question paper
Pgcet Architecture 2018 question paper
 
Pgcet Architecture 2017 question paper
Pgcet Architecture 2017 question paperPgcet Architecture 2017 question paper
Pgcet Architecture 2017 question paper
 
PGCET MBA 2018 question paper
PGCET MBA 2018 question paperPGCET MBA 2018 question paper
PGCET MBA 2018 question paper
 
Civil Service 2019 Prelims Previous Question Paper - 2
Civil Service 2019 Prelims Previous Question Paper - 2Civil Service 2019 Prelims Previous Question Paper - 2
Civil Service 2019 Prelims Previous Question Paper - 2
 
Civil Service 2019 Prelims Previous Question Paper - 1
Civil Service 2019 Prelims Previous Question Paper - 1Civil Service 2019 Prelims Previous Question Paper - 1
Civil Service 2019 Prelims Previous Question Paper - 1
 
Civil Service 2018 Prelims Previous Question Paper - 2
Civil Service 2018 Prelims Previous Question Paper - 2Civil Service 2018 Prelims Previous Question Paper - 2
Civil Service 2018 Prelims Previous Question Paper - 2
 
Civil Service 2018 Prelims Previous Question Paper - 1
Civil Service 2018 Prelims Previous Question Paper - 1Civil Service 2018 Prelims Previous Question Paper - 1
Civil Service 2018 Prelims Previous Question Paper - 1
 
Civil Service 2017 Prelims Previous Question Paper - 2
Civil Service 2017 Prelims Previous Question Paper - 2Civil Service 2017 Prelims Previous Question Paper - 2
Civil Service 2017 Prelims Previous Question Paper - 2
 
Civil Service 2017 Prelims Previous Question Paper - 1
Civil Service 2017 Prelims Previous Question Paper - 1Civil Service 2017 Prelims Previous Question Paper - 1
Civil Service 2017 Prelims Previous Question Paper - 1
 
SNAP 2013 Answer Key
SNAP 2013 Answer KeySNAP 2013 Answer Key
SNAP 2013 Answer Key
 
SNAP 2014 Answer Key
SNAP 2014 Answer KeySNAP 2014 Answer Key
SNAP 2014 Answer Key
 

Recently uploaded

College Call Girls Pune Mira 9907093804 Short 1500 Night 6000 Best call girls...
College Call Girls Pune Mira 9907093804 Short 1500 Night 6000 Best call girls...College Call Girls Pune Mira 9907093804 Short 1500 Night 6000 Best call girls...
College Call Girls Pune Mira 9907093804 Short 1500 Night 6000 Best call girls...Miss joya
 
VIP Call Girls Pune Vrinda 9907093804 Short 1500 Night 6000 Best call girls S...
VIP Call Girls Pune Vrinda 9907093804 Short 1500 Night 6000 Best call girls S...VIP Call Girls Pune Vrinda 9907093804 Short 1500 Night 6000 Best call girls S...
VIP Call Girls Pune Vrinda 9907093804 Short 1500 Night 6000 Best call girls S...Miss joya
 
Kolkata Call Girls Services 9907093804 @24x7 High Class Babes Here Call Now
Kolkata Call Girls Services 9907093804 @24x7 High Class Babes Here Call NowKolkata Call Girls Services 9907093804 @24x7 High Class Babes Here Call Now
Kolkata Call Girls Services 9907093804 @24x7 High Class Babes Here Call NowNehru place Escorts
 
College Call Girls Vyasarpadi Whatsapp 7001305949 Independent Escort Service
College Call Girls Vyasarpadi Whatsapp 7001305949 Independent Escort ServiceCollege Call Girls Vyasarpadi Whatsapp 7001305949 Independent Escort Service
College Call Girls Vyasarpadi Whatsapp 7001305949 Independent Escort ServiceNehru place Escorts
 
Call Girls In Andheri East Call 9920874524 Book Hot And Sexy Girls
Call Girls In Andheri East Call 9920874524 Book Hot And Sexy GirlsCall Girls In Andheri East Call 9920874524 Book Hot And Sexy Girls
Call Girls In Andheri East Call 9920874524 Book Hot And Sexy Girlsnehamumbai
 
Aspirin presentation slides by Dr. Rewas Ali
Aspirin presentation slides by Dr. Rewas AliAspirin presentation slides by Dr. Rewas Ali
Aspirin presentation slides by Dr. Rewas AliRewAs ALI
 
Housewife Call Girls Hoskote | 7001305949 At Low Cost Cash Payment Booking
Housewife Call Girls Hoskote | 7001305949 At Low Cost Cash Payment BookingHousewife Call Girls Hoskote | 7001305949 At Low Cost Cash Payment Booking
Housewife Call Girls Hoskote | 7001305949 At Low Cost Cash Payment Bookingnarwatsonia7
 
Call Girls Hsr Layout Just Call 7001305949 Top Class Call Girl Service Available
Call Girls Hsr Layout Just Call 7001305949 Top Class Call Girl Service AvailableCall Girls Hsr Layout Just Call 7001305949 Top Class Call Girl Service Available
Call Girls Hsr Layout Just Call 7001305949 Top Class Call Girl Service Availablenarwatsonia7
 
Hi,Fi Call Girl In Mysore Road - 7001305949 | 24x7 Service Available Near Me
Hi,Fi Call Girl In Mysore Road - 7001305949 | 24x7 Service Available Near MeHi,Fi Call Girl In Mysore Road - 7001305949 | 24x7 Service Available Near Me
Hi,Fi Call Girl In Mysore Road - 7001305949 | 24x7 Service Available Near Menarwatsonia7
 
Call Girls Hosur Just Call 7001305949 Top Class Call Girl Service Available
Call Girls Hosur Just Call 7001305949 Top Class Call Girl Service AvailableCall Girls Hosur Just Call 7001305949 Top Class Call Girl Service Available
Call Girls Hosur Just Call 7001305949 Top Class Call Girl Service Availablenarwatsonia7
 
Call Girls Kanakapura Road Just Call 7001305949 Top Class Call Girl Service A...
Call Girls Kanakapura Road Just Call 7001305949 Top Class Call Girl Service A...Call Girls Kanakapura Road Just Call 7001305949 Top Class Call Girl Service A...
Call Girls Kanakapura Road Just Call 7001305949 Top Class Call Girl Service A...narwatsonia7
 
Call Girl Koramangala | 7001305949 At Low Cost Cash Payment Booking
Call Girl Koramangala | 7001305949 At Low Cost Cash Payment BookingCall Girl Koramangala | 7001305949 At Low Cost Cash Payment Booking
Call Girl Koramangala | 7001305949 At Low Cost Cash Payment Bookingnarwatsonia7
 
Sonagachi Call Girls Services 9907093804 @24x7 High Class Babes Here Call Now
Sonagachi Call Girls Services 9907093804 @24x7 High Class Babes Here Call NowSonagachi Call Girls Services 9907093804 @24x7 High Class Babes Here Call Now
Sonagachi Call Girls Services 9907093804 @24x7 High Class Babes Here Call NowRiya Pathan
 
Artifacts in Nuclear Medicine with Identifying and resolving artifacts.
Artifacts in Nuclear Medicine with Identifying and resolving artifacts.Artifacts in Nuclear Medicine with Identifying and resolving artifacts.
Artifacts in Nuclear Medicine with Identifying and resolving artifacts.MiadAlsulami
 
Call Girl Indore Vrinda 9907093804 Independent Escort Service Indore
Call Girl Indore Vrinda 9907093804 Independent Escort Service IndoreCall Girl Indore Vrinda 9907093804 Independent Escort Service Indore
Call Girl Indore Vrinda 9907093804 Independent Escort Service IndoreRiya Pathan
 
Call Girls Service Surat Samaira ❤️🍑 8250192130 👄 Independent Escort Service ...
Call Girls Service Surat Samaira ❤️🍑 8250192130 👄 Independent Escort Service ...Call Girls Service Surat Samaira ❤️🍑 8250192130 👄 Independent Escort Service ...
Call Girls Service Surat Samaira ❤️🍑 8250192130 👄 Independent Escort Service ...CALL GIRLS
 
Kesar Bagh Call Girl Price 9548273370 , Lucknow Call Girls Service
Kesar Bagh Call Girl Price 9548273370 , Lucknow Call Girls ServiceKesar Bagh Call Girl Price 9548273370 , Lucknow Call Girls Service
Kesar Bagh Call Girl Price 9548273370 , Lucknow Call Girls Servicemakika9823
 
Call Girls Service Chennai Jiya 7001305949 Independent Escort Service Chennai
Call Girls Service Chennai Jiya 7001305949 Independent Escort Service ChennaiCall Girls Service Chennai Jiya 7001305949 Independent Escort Service Chennai
Call Girls Service Chennai Jiya 7001305949 Independent Escort Service ChennaiNehru place Escorts
 

Recently uploaded (20)

College Call Girls Pune Mira 9907093804 Short 1500 Night 6000 Best call girls...
College Call Girls Pune Mira 9907093804 Short 1500 Night 6000 Best call girls...College Call Girls Pune Mira 9907093804 Short 1500 Night 6000 Best call girls...
College Call Girls Pune Mira 9907093804 Short 1500 Night 6000 Best call girls...
 
VIP Call Girls Pune Vrinda 9907093804 Short 1500 Night 6000 Best call girls S...
VIP Call Girls Pune Vrinda 9907093804 Short 1500 Night 6000 Best call girls S...VIP Call Girls Pune Vrinda 9907093804 Short 1500 Night 6000 Best call girls S...
VIP Call Girls Pune Vrinda 9907093804 Short 1500 Night 6000 Best call girls S...
 
Kolkata Call Girls Services 9907093804 @24x7 High Class Babes Here Call Now
Kolkata Call Girls Services 9907093804 @24x7 High Class Babes Here Call NowKolkata Call Girls Services 9907093804 @24x7 High Class Babes Here Call Now
Kolkata Call Girls Services 9907093804 @24x7 High Class Babes Here Call Now
 
College Call Girls Vyasarpadi Whatsapp 7001305949 Independent Escort Service
College Call Girls Vyasarpadi Whatsapp 7001305949 Independent Escort ServiceCollege Call Girls Vyasarpadi Whatsapp 7001305949 Independent Escort Service
College Call Girls Vyasarpadi Whatsapp 7001305949 Independent Escort Service
 
Call Girls In Andheri East Call 9920874524 Book Hot And Sexy Girls
Call Girls In Andheri East Call 9920874524 Book Hot And Sexy GirlsCall Girls In Andheri East Call 9920874524 Book Hot And Sexy Girls
Call Girls In Andheri East Call 9920874524 Book Hot And Sexy Girls
 
Aspirin presentation slides by Dr. Rewas Ali
Aspirin presentation slides by Dr. Rewas AliAspirin presentation slides by Dr. Rewas Ali
Aspirin presentation slides by Dr. Rewas Ali
 
Housewife Call Girls Hoskote | 7001305949 At Low Cost Cash Payment Booking
Housewife Call Girls Hoskote | 7001305949 At Low Cost Cash Payment BookingHousewife Call Girls Hoskote | 7001305949 At Low Cost Cash Payment Booking
Housewife Call Girls Hoskote | 7001305949 At Low Cost Cash Payment Booking
 
Call Girls Hsr Layout Just Call 7001305949 Top Class Call Girl Service Available
Call Girls Hsr Layout Just Call 7001305949 Top Class Call Girl Service AvailableCall Girls Hsr Layout Just Call 7001305949 Top Class Call Girl Service Available
Call Girls Hsr Layout Just Call 7001305949 Top Class Call Girl Service Available
 
Hi,Fi Call Girl In Mysore Road - 7001305949 | 24x7 Service Available Near Me
Hi,Fi Call Girl In Mysore Road - 7001305949 | 24x7 Service Available Near MeHi,Fi Call Girl In Mysore Road - 7001305949 | 24x7 Service Available Near Me
Hi,Fi Call Girl In Mysore Road - 7001305949 | 24x7 Service Available Near Me
 
Call Girls Hosur Just Call 7001305949 Top Class Call Girl Service Available
Call Girls Hosur Just Call 7001305949 Top Class Call Girl Service AvailableCall Girls Hosur Just Call 7001305949 Top Class Call Girl Service Available
Call Girls Hosur Just Call 7001305949 Top Class Call Girl Service Available
 
Call Girls Kanakapura Road Just Call 7001305949 Top Class Call Girl Service A...
Call Girls Kanakapura Road Just Call 7001305949 Top Class Call Girl Service A...Call Girls Kanakapura Road Just Call 7001305949 Top Class Call Girl Service A...
Call Girls Kanakapura Road Just Call 7001305949 Top Class Call Girl Service A...
 
Call Girl Koramangala | 7001305949 At Low Cost Cash Payment Booking
Call Girl Koramangala | 7001305949 At Low Cost Cash Payment BookingCall Girl Koramangala | 7001305949 At Low Cost Cash Payment Booking
Call Girl Koramangala | 7001305949 At Low Cost Cash Payment Booking
 
Sonagachi Call Girls Services 9907093804 @24x7 High Class Babes Here Call Now
Sonagachi Call Girls Services 9907093804 @24x7 High Class Babes Here Call NowSonagachi Call Girls Services 9907093804 @24x7 High Class Babes Here Call Now
Sonagachi Call Girls Services 9907093804 @24x7 High Class Babes Here Call Now
 
Artifacts in Nuclear Medicine with Identifying and resolving artifacts.
Artifacts in Nuclear Medicine with Identifying and resolving artifacts.Artifacts in Nuclear Medicine with Identifying and resolving artifacts.
Artifacts in Nuclear Medicine with Identifying and resolving artifacts.
 
sauth delhi call girls in Bhajanpura 🔝 9953056974 🔝 escort Service
sauth delhi call girls in Bhajanpura 🔝 9953056974 🔝 escort Servicesauth delhi call girls in Bhajanpura 🔝 9953056974 🔝 escort Service
sauth delhi call girls in Bhajanpura 🔝 9953056974 🔝 escort Service
 
Call Girl Indore Vrinda 9907093804 Independent Escort Service Indore
Call Girl Indore Vrinda 9907093804 Independent Escort Service IndoreCall Girl Indore Vrinda 9907093804 Independent Escort Service Indore
Call Girl Indore Vrinda 9907093804 Independent Escort Service Indore
 
Call Girls Service Surat Samaira ❤️🍑 8250192130 👄 Independent Escort Service ...
Call Girls Service Surat Samaira ❤️🍑 8250192130 👄 Independent Escort Service ...Call Girls Service Surat Samaira ❤️🍑 8250192130 👄 Independent Escort Service ...
Call Girls Service Surat Samaira ❤️🍑 8250192130 👄 Independent Escort Service ...
 
Escort Service Call Girls In Sarita Vihar,, 99530°56974 Delhi NCR
Escort Service Call Girls In Sarita Vihar,, 99530°56974 Delhi NCREscort Service Call Girls In Sarita Vihar,, 99530°56974 Delhi NCR
Escort Service Call Girls In Sarita Vihar,, 99530°56974 Delhi NCR
 
Kesar Bagh Call Girl Price 9548273370 , Lucknow Call Girls Service
Kesar Bagh Call Girl Price 9548273370 , Lucknow Call Girls ServiceKesar Bagh Call Girl Price 9548273370 , Lucknow Call Girls Service
Kesar Bagh Call Girl Price 9548273370 , Lucknow Call Girls Service
 
Call Girls Service Chennai Jiya 7001305949 Independent Escort Service Chennai
Call Girls Service Chennai Jiya 7001305949 Independent Escort Service ChennaiCall Girls Service Chennai Jiya 7001305949 Independent Escort Service Chennai
Call Girls Service Chennai Jiya 7001305949 Independent Escort Service Chennai
 

Gynecology Exam Questions

  • 1. Krok 2 – 2014 Gynecology Base 1. A 52-year-old woman suffering from obesity, complains of bloody discharges from sexual paths during 4 days. Last normal menses were 2 years ago. Histological investigation of biopsy of the endometrium has revealed adenomatous hyperplasia. What reason from the mentioned below caused the development of disease? A. Excessive transformation of preandrogens from adipose tissues B. The increased contents of follicle-stimulating hormone C. Supersecretion of androgens by the cortex of paranephroses. D. Hypersecretion of estrogens by tissues of the organism. E. Poor aromatization of preandrogens due to hypothyroidism 2. A 40-year-old woman complains of colic pains in the lower part of abdomen and abundant bloody discharges from genital tract. Last 2 years she had menses for 15-16 days, abundant, with clots, painful. Had 2 medical abortions. In bimanual investigation: from the canal of the cervix uteri - a fibromatous node, 3 cm in diameter, on the thin stem. Discharges are bloody, moderate.Choose the correct tactics. A. Operation: untwisting of born node B. Phase by phase vitamin therapy C. Hysterectomy without ovaries D. Supravaginal ablation of the uterus without ovaries E. Hormonal hemostasis 3. A 40-year-old woman complains of yellow color discharges from the vagina. Bimanual examination: no pathological changes. Smear test: Trichomonas vaginalis and mixed flora. Colposcopy: two hazy fields on the front labium, with a negative Iodum probing. What is your tactics? A. Treatment of specific colpitis with the subsequent biopsy B. Cervix ectomy C. Cryolysis of cervix uteri D. Diathermocoagulation of the cervix uteri E. Specific treatment of Trichomonas colpitis 4. A 32 y.o. woman consulted a gynecologist about having abundant long menses within 3 months. Bimanual investigation: the body of the uterus is enlarged according to about 12 weeks of pregnancy, distorted, tuberous, of dense consistence. Appendages are not palpated. Histological test of the uterus body mucosa: adenocystous hyperplasia of endometrium. Optimal medical tactics: A. Surgical treatment B. Radial therapy C. Phase by phase vitamin therapy
  • 2. D. Hormonetherapy E. Phytotherapy 5. A woman complains of having slight dark bloody discharges and mild pains in the lower part of abdomen for several days. Last menses were 7 weeks ago. The pregnancy test is positive. Bimanual investigation: the body of the uterus indicates for about 5-6 weeks of pregnancy, it is soft, painless. In the left appendage there is a retort-like formation, 7х5 cm large, mobile, painless. What examination is necessary for detection of fetus localization? A. Ultrasound B. Colposcopy C. Cystoscopy D. Hysteroscopy E. Hromohydrotubation 6. A woman was hospitalised with full-term pregnancy. Examination: the uterus is tender, the abdomen is tense, cardiac tones of the fetus are not auscultated. What is the most probable complication of pregnancy? A. Premature detachment of normally posed placenta B. Premature labor C. Acute hypoxia of a fetus D. Hydramnion E. Back occipital presentation 7. By the end of the 1st period of physiological labor clear amniotic fluid came off. Contractions lasted 35-40 sec every 4-5min. Heartbeat of the fetus was 100 bpm. The BP was 140/90 mm Hg. What is the most probable diagnosis? A. Acute hypoxia of the fetus B. Premature labor C. Back occipital presentation D. Hydramnion E. Premature detachment of normally posed placenta 8. A pregnant woman in her 40th week of pregnancy undergoes obstetric examination: the cervix of uterus is undeveloped. The oxytocin test is negative. Examination at 32 weeks revealed: AP 140/90 mm Hg, proteinuria 1 g/l, peripheral edemata. Reflexes are normal. Choose the most correct tactics: A. Labour stimulation after preparation B. Complex therapy of gestosis for 2 days C. Absolute bed rest for 1 month D. Caesarian section immediately E. Complex therapy of gestosis for 7 days
  • 3. 9. Which gestational age gives the most accurate estimation of weeks of pregnancy by uterine size? A. Less that 12 weeks B. Between 21 and 30 weeks C. Between 12 and 20 weeks D. Between 31 and 40 weeks E. Over 40 weeks 10.A 26 year old woman had the second labour within the last 2 years with oxytocin application. The childs weight is 4080 g. After the placent birth there were massive bleeding, signs of hemorrhagic shock. Despite the injection of contractive agents, good contraction of the uterus and absence of any cervical and vaginal disorders, the bleeding proceeds. Choose the most probable cause of bleeding: A. Atony of the uterus B. Hypotonia of the uterus C. Hysterorrhexis D. Injury of cervix of the uterus E. Delay of the part of placenta 11. A woman is admitted to maternity home with discontinued labor activity and slight bloody discharges from vagina. The condition is severe, the skin is pale, consciousness is confused. BP is 80/40 mm Hg. Heartbeat of the fetus is not heard. There was a Cesarian section a year ago. Could you please determine the diagnosis? A. Hysterorrhesis B. Cord presentation C. Expulsion of the mucous plug from cervix uteri D. Premature expulsion of amniotic fluid E. Placental presentation 12. On the first day after labour a woman had the rise of temperature up to 39°C. Rupture of fetal membranes took place 36 hours before labour. Examination of the bacterial flora of cervix of the uterus revealed hemocatheretic streptococcus of A group. The uterus body is soft, tender. Discharges are bloody, with admixtures of pus. Specify the most probable postnatal complication: A. Metroendometritis B. Infective contamination of the urinary system C. Apostasis of sutures after the episiotomy D. Thrombophlebitis of veins of the pelvis E. Infectious hematoma
  • 4. 13. Rise in temperature up to 39°C was registered the next day after a woman had labor. Fetal membranes rupture took place 36 hours prior to labors. The examination of the bacterial flora of cervix uterirevealed the following: haemolytic streptococcus of group A. The uterus tissue is soft, tender. Discharges are bloody, with mixing of pus. Establish the most probable postnatal complication. A. Metroendometritis B. Infective contamination of the urinary system C. Apostatis of stitches after the episiotomy D. Thrombophlebitis of veins of the pelvis E. Infected hematoma 14. A woman of a high-risk group (chronic pyelonephritis in anamnesis) had vaginal delivery. The day after labour she complained of fever and loin pains, frequent urodynia. Specify the most probable complication: A. Infectious contamination of the urinary system B. Infectious hematoma C. Apostasis of sutures after episiotomy D. Endometritis E. Thrombophlebitis of veins of the pelvis 15. 13 months after the first labor a 24-year-old patient complained of amenorrhea. Pregnancy ended in Caesarian section because of premature detachment of normally positioned placenta which resulted in blood loss at the rate of 2000 ml owing to disturbance of blood clotting. Choose the most suitable investigation: A. Estimation of gonadotropin rate B. Progesteron assay C. USI of small pelvis organs D. Computer tomography of head E. Estimation of testosteron rate in blood serum 16. In 13 months after the first labor a 24-year-old woman complains of amenorrhea. Cesarian section was conducted as a result of premature detachment of normally posed placenta. Hemorrhage has made low fidelity of 2000 ml due to breakdown of coagulation of blood. Choose the most suitable investigation. A. Determination of the level of gonadotropin B. Progesteron test C. Determination of the contents of testosteron-depotum in blood serum. D. Computer tomography of the head E. Ultrasound of organs of a small pelvis
  • 5. 17. In the woman of 24 years about earlier normal menstrual function, cycles became irregular, according to tests of function diagnostics - anovulatory. The contents of prolactin in blood is boosted. Choose the most suitable investigation: A. Computer tomography of the head B. USI of organs of small pelvis C. Determination of the contents of testosteron-depotum in blood serum D. Progesterone assay E. Determination of the level of gonadotropins 18. A woman in her 39th week of pregnancy, the second labour, has regular birth activity. Uterine contractions take place every 3 minutes. What criteria describe the beginning of the II labor stage the most precisely? A. Cervical dilatation by no less than 4 cm B. Cervical smoothing over 90% C. Presenting part is in the lower region of small pelvis D. Rupture of fetal bladder E. Duration of uterine contractions over 30 seconds 19. A 20-year-old woman is having timed labor continued for 4 hours. Light amniotic fluid came off. The fetus head is pressed to the orifice in the small pelvis. The anticipated fetus mass is 4000,0 gpm 200,0 g. Heartbeat of the fetus is normal. Intrinsic examination: cervix is absent, disclosure – 2 cm, the fetal membranes are not present. The head is in 1-st plane of the pelvis, a sagittal suture is in the left slanting dimension. What is the purpose of glucose-calcium-hormone - vitaminized background conduction? A. Prophylaxes of weakness of labor activity B. Fetus hypoxia prophylaxes C. Treatment of weakness of labor activity. D. Antenatal preparation E. Labor stimulation 20.A 24 years old primipara was hospitalised with complaints about discharge of the amniotic waters. The uterus is tonic on palpation. The position of the fetus is longitudinal, it is pressed with the head to pelvic outlet. Palpitation of the fetus is rhythmical, 140 bpm, auscultated on the left below the navel. Internal examination: cervix of the uterus is 2,5 cm long, dense, the external os is closed, light amniotic waters out of it. Point a correct component of the diagnosis: A. Antenatal discharge of the amniotic waters B. Early discharge of the amniotic waters C. The end of the 1st stage of labour
  • 6. D. Pathological preterm labour E. The beginning of the 1st stage of labour 21. A 34 y.o. woman in her 29-th week of pregnancy, that is her 4-th labor to come, was admitted to the obstetric department with complaints of sudden and painful bloody discharges from vagina that appeared 2 hours ago. The discharges are profuse and contain grumes. Cardiac funnction of the fetus is rhytmic, 150 strokes in the minute, uterus tone is normal. The most probable provisional diagnosis will be: A. Placental presentation B. Disseminated intravascular coagulation syndrome C. Vasa previa D. Detachment of normally located placenta E. Bloody discharges 22.A 29 year old patient underwent surgical treatment because of the benign serous epithelial tumour of an ovary. The postoperative period has elapsed without complications. What is it necessary toprescribe for the rehabilitational period: A. Hormonotherapy and proteolytic enzymes B. Magnitotherapy and vitamin therapy C. The patient does not require further care D. Antibacterial therapy and adaptogens E. Lasertherapy and enzymotherapy 23.A 34-year-old woman with 10-week pregnancy (the second pregnancy) has consulted gynaecologist to make a record in patient chart. There was a hydramnion previous pregnancy, the birth weight of a child was 4086 g. What tests are necessary first of all? A. The test for tolerance to glucose B. Fetus cardiophonography C. Ultrasound of the fetus D. Determination of the contents of alpha fetoprotein E. Bacteriological test of discharge from the vagina 24.A 26 y.o. woman complains of sudden pains in the bottom of abdomen irradiating to the anus, nausea, giddiness, bloody dark discharges from sexual tracts for one week, the delay of menses for 4 weeks. Signs of the peritoneum irritation are positive. Bimanual examination: borders of the uterus body and its appendages are not determined because of sharp painfullness. The diverticulum and painfullness of the back and dextral fornixes of the vagina are evident. What is the most probable diagnosis? A. Broken tubal pregnancy
  • 7. B. Acute right-side adnexitis C. Apoplexy of the ovary D. Torsion of the crus of the ovary tumour E. Acute appendicitis 25.At the gynaecological department there is a patient of 32 years with the diagnosis: "acute bartholinitis". Body temperature is 38,2°C, leucocytes count 10,4×109/L, the ESR is 24 mm/hour. In the area of big gland of the vestibulum - the dermahemia, the sign of the fluctuation, sharp tenderness (pain). What is the most correct tactics of the doctor? A. Surgical dissecting, a drainage of an abscess of the gland, antibiotics B. Antibiotics, Sulfanilamidums C. Antibiotic therapy D. Antibiotics, detoxication and biostimulants. E. Surgical dissection, drainage of the abscess of the gland 26.A primagravida with pregnancy of 37-38 weeks complains of headache, nausea, pain in epigastrium. Objective: the skin is acyanotic. Face is hydropic, there is short fibrillar twitching of blepharons, muscles of the face and the inferior extremities. The look is fixed. AP- 200/110 mm Hg; sphygmus of 92 bpm, intense. Respiration rate is 32/min. Heart activity is rhythmical. Appreciable edemata of the inferior extremities are present. Urine is cloudy. What medication should be administered? A. Droperidolum of 0,25% - 2,0 ml B. Dibazolum of 1% - 6,0 ml C. Hexenalum of 1% - 2,0 ml D. Pentaminum of 5% - 4,0 ml E. Papaverine hydrochloride of 2% - 4,0 ml 27. An onset of severe preeclampsia at 16 weeks gestation might be caused by: A. Hydatidiform mole B. Anencephaly C. Maternal renal disease D. Interventricular defect of the fetus E. Twin gestation 28.A woman had the rise of temperature up to 39°C on the first day after labour. The rupture of fetal membranes took place 36 hours before labour. The investigation of the bacterial flora of cervix of the uterus revealed hemocatheretic streptococcus of group A. The uterus body is soft, tender.
  • 8. Discharges are bloody, mixed with pus. Specify the most probable postnatal complication: A. Metroendometritis B. Infection of the urinary system C. Apostatis of junctures after the episiotomy D. Thrombophlebitis of pelvic veins E. Infected hematoma 29.A 24 y.o. patient 13 months after the first labour consulted a doctor about amenorrhea. Pregnancy has concluded by a Cesarean section concerning to a premature detachment of normally posed placenta hemorrhage has made low fidelity 2000 ml owing to breakdown of coagulability of blood. Choose the most suitable investigation: A. Determination of the level of Gonadotropins B. Progesteron assay C. USI of organs of a small pelvis D. Computer tomography of the head E. Determination of the contents of Testosteron-Depotum in Serum of blood 30.A 34 year old woman in the 10th week of gestation (the second pregnancy) consulted a doctor of antenatal clinic in order to be registered there. In the previous pregnancy hydramnion was observed, the childs birth weight was 4086 g. What examination method should be applied in the first place? A. The test for tolerance to glucose B. US of fetus C. Bacteriological examination of discharges from vagina D. Determination of the contents of fetoproteinum E. A cardiophonography of fetus 31. A 10 y.o. boy was ill with angina 2 weeks ago, has complaints of joint pain and stiffness of his left knee and right elbow. There was fever (38,50) and ankle disfunction, enlargement of cardiac dullness by 2 cm, tachycardia, weakness of the 1st sound, gallop rhythm, weak systolic murmur near apex. What diagnosis corresponds with such symptoms? A. Acute rheumatic fever B. Reiters disease C. Reactive arthritis D. Systemic lupus erythematosis E. Juvenile rheumatoid arthritis 32.The disease began acutely. The frequent watery stool developed 6 hours ago. The bodys temperature is normal. Then the vomiting was joined. On examination: his voice is hoarse, eyes are deeply sunken in the orbits. The
  • 9. pulse is frequent. Blood pressure is low. There is no urine. What is the preliminary diagnosis? A. Cholera B. Typhoid fever C. Salmonellosis D. Toxic food-borne infection E. Dysentery 33.At term of a gestation of 40 weeks height of standing of a uterine fundus is less then assumed for the given term. The woman has given birth to the child in weight of 2500 g, a length of a body 53 cm, with an assessment on a scale of Apgar of 4-6 points. Labor were fast. The cause of such state of the child were: A. Chronic fetoplacental insufficiency B. Delay of an intra-uterine fetation C. Infection of a fetus D. Prematurity E. Placental detachment 34.A pregnant woman may be diagnosed with hepatitis if it is confirmed by the presence of elevated: A. SGOT (ALT) B. WBCs C. Sedimentation rates D. Alkaline phosphatase E. BUN 35.A woman, aged 40, primigravida, with infertility in the medical history, on the 42-43 week of pregnancy. Labour activity is weak. Longitudinal presentation of the fetus, I position, anterior position. The head of the fetus is engaged to pelvic inlet. Fetus heart rate is 140 bmp, rhythmic, muffled. Cervix dilation is 4 cm. On amnioscopy: greenish colour of amniotic fluid and fetal membranes. Cranial bones are dense, cranial sutures and small fontanel are diminished. What should be tactics of delivery? A. Caesarean section B. Medication sleep, amniotomy, labour stimulation C. Fetal hypoxia treatment, in the II period - forceps delivery D. Amniotomy, labour stimulation, fetal hypoxia treatment E. Fetal hypoxia treatment, conservative delivery 36.An endometrial adenocarcinoma that has extended tothe uterine serosa would be classified as stage: A. IIIA B. IIA
  • 10. C. IC D. IIB E. IVAB 37. Which of the methods of examination is the most informative in the diagnostics of a tube infertility? A. Laparoscopy with chromosalpingoscopy B. Hysterosalpingography C. Pertubation D. Transvaginal echography E. Bicontrast pelviography 38.A pregnant woman (35 weeks), aged 25, was admitted to the hospital because of bloody discharges. In her medical history there were two artificial abortions. In a period of 28-32 weeks there was noted the onset of hemorrhage and USD showed a placental presentation. The uterus is in normotonus, the fetus position is transversal (Ist position). The heartbeats is clear, rhythmical, 140 bpm. What is the further tactics of the pregnant woman care? A. To perform a delivery by means of Cesarean section B. Tointroduct the drugs to increase the blood coagulation and continue observation C. Tokeep the intensity of hemorrhage under observation and after the bleeding is controlled to prolong the pregnancy D. Stimulate the delivery by intravenous introduction of oxytocin E. Toperform the hemotransfusion and to prolong the pregnancy 39.A woman, primagravida, consults a gynecologist on 05.03.2012. A week ago she felt the fetus movements for the first time. Last menstruation was on 10.01.2012. When should she be given maternity leave? A. 8 August B. 22 August C. 5 September D. 11 July E. 25 July 40.Condition of a parturient woman has been good for 2 hours after live birth: uterus is thick, globe-shaped, its bottom is at the level of umbilicus, bleeding is absent. The clamp put on the umbilical cord remains at the same level, when the woman takes a deep breath or she is being pressed over the symphysis with the verge of hand, the umbilical cord drows into the vagina. Bloody discharges from the sexual tracts are absent. What is the doctors further tactics?
  • 11. A. To do manual removal of afterbirth B. Todo curettage of uterine cavity C. Tointroduct oxitocine intravenously D. Toapply Abduladze method E. Toapply Credes method 41. The woman who has delivered twins has early postnatal hypotonic uterine bleeding reached 1,5% of her bodyweight. The bleeding is going on. Conservative methods to arrest the bleeding have been found ineffective. The conditions of patient are pale skin, acrocyanosis, oliguria. The woman is confused. The pulse is 130 bpm, BP– 75/50 mm Hg. What is the further treatment? A. Uterine extirpation B. Uterine vessels ligation C. Putting clamps on the uterine cervix D. Inner glomal artery ligation E. Supravaginal uterine amputation 42.A 26 y.o. woman complains of a mild bloody discharge from the vagina and pain in the lower abdomen. She has had the last menstruation 3,5 months ago. The pulse is 80 bpm. The blood pressure (BP) is 110/60 mm Hg and body temperature is 36,6°C. The abdomen is tender in the lower parts. The uterus is enlarged up to 12 weeks of gestation. What is your diagnosis? A. Inevitable abortion B. Incomplete abortion C. Disfunctional bleeding D. Complete abortion E. Incipient abortion 43.18 y.o. woman complains of pain in the lower abdomen. Some minutes before she has suddenly appeared unconscious at home. The patient had no menses within last 3 months. On examination: pale skin, the pulse- 110 bpm, BP- 80/60 mm Hg. The Schyotkins sign is positive. Hb- 76 g/L. The vaginal examination: the uterus is a little bit enlarged, its displacement is painful. There is also any lateral swelling of indistinct size. The posterior fornix of the vagina is tendern and overhangs inside. What is the most probable diagnosis? A. Impaired extrauterine pregnancy B. Acute appendicitis C. Twist of cystoma of right uterine adnexa D. Ovarian apoplexy E. Acute salpingoophoritis
  • 12. 44.A 20 y.o. pregnant woman with 36 weeks of gestation was admitted to the obstetrical hospital with complains of pain in the lower abdomen and bloody vaginal discharge. The general condition of the patient is good. Her blood pressure is 120/80 mm Hg. The heart rate of the fetus is 140 bpm, rhythmic. Vaginal examination: the cervix of the uterus is formed and closed. The discharge from vagina is bloody up to 200 ml per day. The head of the fetus is located high above the minor pelvis entry. A soft formation was defined through the anterior fornix of the vagina. What is the probable diagnosis? A. Placental presentation B. Premature placental separation C. Threatened premature labor D. Incipient abortion E. Uterine rupture 45.In the gynecologic office a 28 y.o. woman complains of sterility within three years. The menstrual function is not impaired. There were one artificial abortion and chronic salpingo-oophoritis in her case history. Oral contraceptives were not used. Her husbands analysis of semen is without pathology. What diagnostic method will you start from the workup in this case of sterility? A. Hysterosalpingography B. Ultra sound investigation C. Hysteroscopia D. Diagnostic scraping out of the uterine cavity E. Hormone investigation 46.A 28-year-old patient underwent endometrectomy as a result of incomplete abortion. Blood loss was at the rate of 900 ml. It was necessary tostart hemotransfusion. After transfusion of 60 ml of erythrocytic mass the patient presented with lumbar pain and fever which resulted in hemotransfusion stoppage. 20 minutes later the patients condition got worse: she developed adynamia, apparent skin pallor, acrocyanosis, profuse perspiration. to- 38,5°C, Ps- 110/min, AP- 70/40 mm Hg. What is the most likely diagnosis? A. Hemotransfusion shock B. DIC syndrome C. Septic shock D. Hemorrhagic shock E. Anaphylactic shock 47. A 58-year-old female patient came to the antenatal clinic complaining of bloody light-red discharges from the genital tracts. Menopause is 12 years. Gynaecological examination revealed age involution of externalia and vagina; uterine cervix was unchanged, there were scant bloody discharges from
  • 13. uterine cervix, uterus was of normal size; uterine appendages were not palpable; parametria were free. What is the most likely diagnosis? A. Uterine carcinoma B. Cervical carcinoma C. Granulosa cell tumor of ovary D. Atrophic colpitis E. Abnormalities of menstrual cycle of climacteric nature 48.The results of a separate diagnostic curettage of the mucous of the uterus cervix and body made up in connection with bleeding in a postmenopausal period: the scrape of the mucous of the cervical canal revealed no pathology, in endometrium - the highly differentiated adenocarcinoma was found. Metastases are not found. What method of treatment is the most correct? A. Surgical treatment and hormonotherapy B. Surgical treatment and radial therapy C. Surgical treatment + chemotherapy D. Radial therapy 49.A 27 y.o. woman complains of having the disorders of menstrual function for 3 months, irregular pains in abdomen. On bimanual examination: in the dextral appendage range of uterus there is an elastic spherical formation, painless, 7 cm in diameter. USI: in the right ovary - a fluid formation, 4 cm in diameter, unicameral, smooth. What method of treatment is the most preferable? A. Prescription of an estrogen-gestogen complex for 3 months with repeated examination B. Dispensary observation of the patient C. Chemotherapeutic treatment D. Anti-inflammatory therapy E. Operative treatment 50.A 40 year old patient complains of yellowish discharges from the vagina. Bimanual examination revealed no pathological changes. The smear contains Trichomonas vaginalis and blended flora. Colposcopy revealed two hazy fields on the frontal labium, with a negative Iodine test. Your tactics: A. Treatment of specific colpitis and subsequent biopsy B. Cervix ectomy C. Cryolysis of cervix of the uterus D. Diathermocoagulation of the cervix of the uterus E. Specific treatment of Trichomonas colpitis 51. A 26-year-old secundipara at 40 weeks of gestation arrived at the maternity ward after the beginning of labor activity. 2 hours before, bursting of waters occurred. The fetus was in a longitudinal lie with cephalic presentation.
  • 14. Abdominal circumference was 100 cm, fundal height - 42 cm. Contractions occurred every 4-5 minutes and lasted 25 seconds each. Internal obstetric examination revealed cervical effacement, opening by 4 cm. Fetal bladder was absent. Fetal head was pressed against the pelvic inlet. What complication arose in childbirth? A. Early amniorrhea B. Secondary uterine inertia C. Clinically narrow pelvis D. Discoordinated labor E. Primary uterine inertia 52.A 28-year-old parturient complains about headache, vision impairment, psychic inhibition. Objectively: AP- 200/110mm Hg, evident edemata of legs and anterior abdominal wall. Fetus head is in the area of small pelvis. Fetal heartbeats is clear, rhythmic, 190/min. Internal examination revealed complete cervical dilatation, fetus head was in the area of small pelvis. What tactics of labor management should be chosen? A. Forceps operation B. Cesarean C. Conservative labor management with episiotomy D. Stimulation of labor activity E. Embryotomy 53.A 28 year old woman had the second labour and born a girl with manifestations of anemia and progressing jaundice. The childs weight was 3 400 g, the length was 52 cm. The womans blood group is B (III) Rh-, the fathers blood group is A (III) Rh+, the childs blood group is B (III) Rh+. What is the cause of anemia? A. Rhesus incompatibility B. Antigen A incompatibility C. Antigen AB incompatibility D. Intrauterine infection E. Antigen B incompatibility 54.A 48 year old female patient complains about contact haemorrhage. Speculum examination revealed hypertrophy of uterus cervix. It resembles of cauliflower, it is dense and can be easily injured. Bimanual examination revealed that fornices were shortened, uterine body was nonmobile. What is the most probable diagnosis? A. Cervical carcinoma B. Metrofibroma C. Cervical pregnancy D. Cervical papillomatosis E. Endometriosis
  • 15. 55.A 37 y.o. primigravida woman has been having labor activity for 10 hours. Labor pains last for 20-25 seconds every 6-7 minutes. The fetus lies in longitude, presentation is cephalic, head is pressed upon the entrance to the small pelvis. Vaginal examination results: cervix of uterus is up to 1 cm long, lets 2 transverse fingers in. Fetal bladder is absent. What is the most probable diagnosis? A. Primary uterine inertia B. Discoordinated labor activity C. Pathological preliminary period D. Secondary uterine inertia E. Normal labor activity 56.Laparotomy was performed to a 54 y.o. woman on account of big formation in pelvis that turned out to be one-sided ovarian tumor along with considerable omental metastases. The most appropriate intraoperative tactics involves: A. Ablation of omentum, uterus and both ovaries with tubes B. Biopsy of omentum C. Ablation of an ovary and omental metastases D. Ablation of omentum and both ovaries with tubes E. Biopsy of an ovary 57. A parturient complains about pain in the mammary gland. Palpation revealed a 3×4 cm large infiltration, soft in the centre. Body temperature is 38,5°C. What is the most probable diagnosis? A. Acute purulent mastitis B. Pleuritis C. Birth trauma D. Retention of milk E. Pneumonia 58.A 43 y.o. patient complains of formation and pain in the right mammary gland, rise of temperature up to 37,2°C during the last 3 months. Condition worsens before the menstruation. On examination: edema of the right breast, hyperemia, retracted nipple. Unclear painful infiltration is palpated in the lower quadrants. What is the most probable diagnosis? A. Cancer of the right mammary gland B. Tuberculosis of the right mammary gland C. Right-side chronic mastitis D. Right-side acute mastitis E. Premenstrual syndrome
  • 16. 59.A 14 year old girl complains of profuse bloody discharges from genital tracts during 10 days after suppresion of menses for 1,5 month. Similiar bleedings recur since 12 years on the background of disordered menstrual cycle. On rectal examination: no pathology of the internal genitalia. In blood: Hb - 70 g/l, RBC- 2,3×1012/l, Ht - 20. What is the most probable diagnosis? A. Juvenile bleeding, posthemorrhagic anemia B. Polycyst ovarian syndrome C. Werlholfs disease D. Hormonoproductive ovary tumor E. Incomplete spontaneous abortion 60.A 33-year-old woman was urgently brought to clinic with complaints of the pain in the lower part of the abdomen, mostly on the right, irradiating to rectum, she also felt dizzy. The above mentioned complaints developed acutely at night. Last menses were 2 weeks ago. On physical exam: the skin is pale, Ps - 92 bpm, t- 36,6°C, BP- 100/60 mm Hg. The abdomen is tense, slightly tender in lower parts, peritoneal symptoms are slightly positive. Hb- 98 g/L. What is the most probable diagnosis? A. Apoplexy of the ovary B. Renal colic C. Intestinal obstruction D. Acute appendicitis E. Abdominal pregnancy 61. A secundipara has regular birth activity. Three years agoshe had cesarean section for the reason of acute intrauterine hypoxia. During parodynia she complains of extended pain in the area of postsurgical scar. Objectively: fetus pulse is rhythmic - 140 bpm. Vaginal examination shows 5 cm cervical dilatation. Fetal bladder is intact. What is the tactics of choice? A. Cesarean section B. Obstetrical forceps C. Augmentation of labour D. Waiting tactics of labor management E. Vaginal delivery 62.A 54-year-old female patient consulted a doctor about bloody discharges from the genital tracts after 2 years of amenorrhea. USI and bimanual examination revealed no genital pathology. What is the tactics of choice? A. Fractional biopsy of lining of uterus and uterine mucous membranes B. Contracting drugs C. Hysterectomy D. Estrogenic haemostasia E. Styptic drugs
  • 17. 63.Examination of a just born placenta reveals defect 2x3 cm large. Hemorrhage is absent. What tactic is the most reasonable? A. Manual uretus cavity revision B. Prescription of uterotonic medicines C. Parturient supervision D. Instrumental uterus cavity revision E. External uterus massage 64.A 27 y.o. gravida with 17 weeks of gestation was admitted to the hospital. There was a history of 2 spontaneous miscarriages. On bimanual examination: uterus is enlarged to 17 weeks of gestation, uterus cervix is shortened, isthmus allows to pass the finger tip. The diagnosis is isthmico-cervical insufficiency. What is the doctors tactics? A. To place suture on the uterus cervix B. Toperform amniocentesis C. Tointerrupt pregnancy D. Toadminister tocolytic therapy E. Toadminister hormonal treatment 65.A 27-year-old woman presents at the maternity welfare centre because of infertility. She has had sexual life in marriage for 4 years, doesnt use contraceptives. She hasnt get pregnant. On examination: genital development is without pathology, uterine tubes are passable, basal (rectal) temperature is one-phase during last 3 menstrual cycles. What is the infertility cause? A. Anovular menstrual cycle B. Abnormalities in genital development C. Chronic adnexitis D. Immunologic infertility E. Genital endometriosis 66.A 43 y.o. woman complains of contact hemorrhages during the last 6 months. Bimanual examination: cervix of the uterus is enlarged, its mobility is reduced. Mirrors showed the following: cervix of the uterus is in the form of cauliflower. Chrobak and Schiller tests are positive. What is the most probable diagnosis? A. Cancer of cervix of the uterus B. Cervical pregnancy C. Leukoplakia D. Nascent fibroid E. Polypus of the cervis of the uterus
  • 18. 67. A 26-year-old woman gave birth to a child 6 months ago. She applied to gynecologist complaining of menstruation absence. The child is breast-fed. Vagina exam: uterus is of normal form, dense consistence. What is the most probable diagnosis? A. Physiological amenorrhea B. Sheehans syndrome C. Gestation D. Pseudoamenorrhea E. Ashermans syndrome 68.A primagravida in her 20th week of gestation complains about pain in her lower abdomen, blood smears from the genital tracts. The uterus has an increased tonus, the patient feels the fetus movements. Bimanual examination revealed that the uterus size corresponded the term of gestation, the uterine cervix was contracted down to 0,5 cm, the external orifice was open by 2 cm. The discharges were bloody and smeary. What is the most likely diagnosis? A. Incipient abortion B. Abortion in progress C. Missed miscarriage D. Incomplete abortion E. Risk of abortion 69.Full-term pregnancy. Body weight of the pregnant woman is 62 kg. The fetus has the longitudinal position, the fetal head is pressed against the pelvic inlet. Abdominal circumference is 100 cm. Fundal height is 35 cm. What is the approximate weight of the fetus? A. 3 kg 500 g B. 4 kg C. 3 kg D. 4 kg 500 g E. 2 kg 500 g 70.A patient was admitted to the hospital with complaints of periodical pain in the lower part of abdomen that gets worse during menses, weakness, malaise, nervousness, dark bloody smears from vagina directly before and after menses. Bimanual examination revealed that uterus body is enlarged, appendages cannot be palpated, posterior fornix has tuberous surface. Laparoscopy revealed: ovaries, peritoneum of rectouterine pouch and pararectal fat have "cyanotic eyes". What is the most probable diagnosis? A. Disseminated form of endometriosis B. Chronic salpingitis C. Ovarian cystoma
  • 19. D. Tuberculosis of genital organs E. Polycystic ovaries 71. A gravida with 7 weeks of gestation is referred for the artificial abortion. On operation while dilating cervical canal with Hegar dilator No.8 a doctor suspected uterus perforation. What is immediate doctors tactics to confirm the diagnosis? A. Probing of uterus cavity B. Ultrasound examination C. Bimanual examination D. Laparoscopy E. Metrosalpingography 72. A pregnant woman in her 8th week was admitted to the hospital for artificial abortion. In course of operation during dilatation of cervical canal of uterus by means of Hegars dilator No.8 the doctor suspected uterus perforation. What is the immediate tactics for confirmation of this diagnosis? A. Uterine probing B. US examination C. Metrosalpingography D. Laparoscopy E. Bimanual examination 73. A 59 year old female patient applied to a maternity welfare clinic and complained about bloody discharges from the genital tracts. Postmenopause is 12 years. Vaginal examination revealed that external genital organs had signs of age involution, uterus cervix was not erosive, small amount of bloody discharges came from the cervical canal. Uterus was of normal size, uterine appendages were unpalpable. Fornices were deep and painless. What method should be applied for the diagnosis specification? A. Separated diagnosic curretage B. Culdoscopy C. Puncture of abdominal cavity through posterior vaginal fornix D. Laparoscopy E. Extensive colposcopy 74. A 25-year-old woman complains of profuse foamy vaginal discharges, foul, burning and itching in genitalia region. She has been ill for a week. Extramarital sexual life. On examination: hyperemia of vaginal mucous, bleeding on touching, foamy leucorrhea in the urethral area. What is the most probable diagnosis? A. Trichomonas colpitic B. Bacterial vaginosis
  • 20. C. Chlamydiosis D. Gonorrhea E. Vagina candidomicosis 75. A 26 year old woman who delivered a child 7 months ago has been suffering from nausea, morning vomiting, sleepiness for the last 2 weeks. She suckles the child, menstruation is absent. She hasnt applied any contraceptives. What method should be applied in order to specify her diagnosis? A. Ultrasonic examination B. Speculum examination C. Palpation of mammary glands and pressing-out of colostrum D. Roentgenography of small pelvis organs E. Bimanual vaginal examination 76. A newborns head is of dolichocephalic shape, that is front-to-back elongated. Examination of the occipital region revealed a labour tumour located in the middle between the prefontanel and posterior fontanel. Specify the type of fetal presentation: A. Posterior vertex presentation B. Presentation of the bregma C. Face presentation D. Brow presentation E. Anterior vertex presentation 77. A woman consulted a doctor on the 14th day after labour about sudden pain, hyperemy and induration of the left mammary gland, body temperature rise up to 39°C, headache, indisposition. Objectively: fissure of nipple, enlargement of the left mammary gland, pain on palpation. What pathology would you think about in this case? A. Lactational mastitis B. Phlegmon of mammary gland C. Fibrous adenoma of the left mammary gland D. Lacteal cyst with suppuration E. Breast cancer 78. A young woman applied to gynecologist due to her pregnancy of 4-5 weeks. The pregnancy is desirable. Anamnesis stated that she had rheumatism in the childhood. Now she has combined mitral heart disease with the priority of mitral valve deficiency. When will she need the inpatient treatment (what periods of pregnancy)? A. 8-12 weeks, 28–32 weeks, 37 weeks B. 12-16 weeks, 27-28 weeks, 37-38 weeks C. 16 weeks, 34 weeks, 39-40 weeks
  • 21. D. 6-7weeks, 16 weeks, 38 weeks E. 10-12 weeks, 24 weeks, 37-38 weeks 79. A woman in the first half of pregnancy was brought to clinic by an ambulance. Term of pregnancy is 36 weeks. She complains of intensive pain in the epigastrium, had vomiting for 2 times. Pain started after the patient had eaten vinaigrette. Swelling of lower extremities. BP - 140/100 mm Hg. Urine became curd after boiling. What is the most probable diagnosis? A. Preeclampsia B. Food toxicoinfection C. Exacerbation of pyelonephritis D. Dropsy of pregnant women E. Nephropathy of the 3rd degree 80.A 13 year old girl consulted the school doctor on account of moderate bloody discharge from the genital tracts, which appeared 2 days ago. Secondary sexual characters are developed. What is the most probable cause of bloody discharge? A. Menarche B. Werlhofs disease C. Haemophilia D. Juvenile hemorrhage E. Endometrium cancer 81. In 10 min after childbirth by a 22-year-old woman, the placenta was spontaneousely delivered and 100 ml of blood came out. Woman weight - 80 kg, infant weight - 4100 g, length - 53 cm. The uterus contracted. In 10 minutes the hemorrhage renewed and the amount of blood constitued 300 ml. What amount of blood loss is permissible for this woman? A. 400 ml B. 650 ml C. 300 ml D. 1000 ml E. 500 ml 82.A pregnant woman was registered in a maternity welfare clinic in her 11th week of pregnancy. She was being under observation during the whole term, the pregnancy course was normal. What document must the doctor give the pregnant woman to authorize her hospitalization in maternity hospital? A. Exchange card B. Medical certificate C. Sanitary certificate D. Appointment card for hospitalization
  • 22. E. Individual prenatal record 83.After examination a 46-year-old patient was diagnosed with left breast cancer T2N2M0, cl. gr. II-a. What will be the treatment plan for this patient? A. Radiation therapy + operation + chemotherapy B. Operation only C. Radiation therapy only D. Chemotherapy only E. Operation + radiation therapy 84.Immediately after delivery a woman had haemorrhage, blood loss exceeded postpartum haemorrhage rate and was progressing. There were no symptoms of placenta detachment. What tactics should be chosen? A. Manual removal of placenta and afterbirth B. Instrumental revision of uterine cavity walls C. Intravenous injection of methylergometrine with glucose D. Removal of afterbirth by Credes method E. Uterus tamponade 85.A 30 y.o. primigravida woman has got intensive labor pain every 1-2 minutes that lasts 50 seconds. The disengagement has started. The perineum with the height of 4 cm has grown pale. What actions are necessary in this situation? A. Episiotomy B. Expectant management C. Perineotomy D. Perineum protection E. Vacuum extraction of fetus 86.A 30-year-old gravida consulted a gynecologist about bright red bloody discharges from the vagina in the 32 week of gestation. She was hospitalized with a suspicion of placental presentation. Under what conditions is it rational to conduct the internal examination in order to make a diagnosis? A. In the operating room prepared for the operation B. In the admission ward of maternity hospital C. The examination is not to be conducted because of risk of profuse haemorrhage D. In the delivery room keeping to all the aseptics regulations E. In the examination room of antenatal clinic
  • 23. 87. A 28 y.o. primagravida, pregnancy is 15-16 weaks of gestation, presents to the maternity clinics with dull pain in the lower part of the abdomen and in lumbar area. On vaginal examination: uterus cervix is 2,5 cm, external isthmus allows to pass the finger tip. Uterus body is enlarged according to the pregnancy term. Genital discharges are mucous, mild. What is the diagnosis? A. Threatened spontaneous abortion B. Hydatid molar pregnancy C. Placenta presentation D. Spontaneous abortion which has begun E. Stopped pregnancy 88.A primapara with pelvis size 25-28-31-20 cm has active labor activity. Waters poured out, clear. Fetus weight is 4500 g, the head is engaged to the small pelvis inlet. Vastens sign as positive. Cervix of uterus is fully dilated. Amniotic sac is absent. The fetus heartbeat is clear, rhythmic, 136 bpm. What is the labor tactics? A. Caesarean section B. Obstetrical forseps C. Vacuum extraction of the fetus D. Conservative tactics of labor E. Stimulation of the labor activity 89.Internal obstetric examination of a parturient woman revealed that the sacrum hollow was totally occupied with fetus head, ischiadic spines couldnt be detected. Sagittal suture is in the straight diameter, occipital fontanel is directed towards symphysis. In what plane of small pelvis is the presenting part of the fetus? A. Plane of pelvic outlet B. Wide pelvic plane C. Plane of pelvic inlet D. Over the pelvic inlet E. Narrow pelvic plane 90.A 30 y.o. woman has the 2-nd labour that has been lasting for 14 hours. Hearbeat of fetus is muffled, arrhythmic, 100/min. Vaginal examination: cervix of uterus is completely opened, fetus head is level with outlet from small pelvis. Saggital suture is in the straight diameter, small crown is near symphysis. What is the further tactics of handling the delivery? A. Use of obstetrical forceps B. Use of cavity forceps C. Cesarean section D. Stimulation of labour activity by oxytocin E. Cranio-cutaneous (Ivanovs) forceps
  • 24. 91. During examination of a patient, masses in the form of condyloma on a broad basis are found in the area of the perineum. What is the tactics of the doctor? A. To send a woman into dermatological and venerological centre B. Cryodestruction of condyloms C. Chemical coagulator treatment D. Antiviral treatment E. Surgical ablation of condyloms 92.A woman at 30 weeks pregnant has had an attack of eclampsia at home. On admission to the maternity ward AP is 150/100 mm Hg. Predicted fetal weight is 1500 g. There is face and shin pastosity. Urine protein is 0,66o/oo. Parturient canal is not ready for delivery. An intensive complex therapy has been started. What is the correct tactics of this case management? A. Delivery by cesarean section B. Continue therapy and prolong pregnancy for 3-4 weeks C. Treat preeclampsia and achieve the delivery by way of conservative management D. Labor induction by intravenous oxytocin or prostaglandins E. Continue therapy and prolong pregnancy for 1-2 weeks 93.A 28 year old woman has bursting pain in the lower abdomen during menstruation; chocolate-like discharges from vagina. It is known from the anamnesis that the patient suffers from chronic adnexitis. Bimanual examination revealed a tumour-like formation of heterogenous consistency 7*7 cm large to the left from the uterus. The formation is restrictedly movable, painful when moved. What is the most probable diagnosis? A. Endometrioid cyst of the left ovary B. Fibromatous node C. Tumour of sigmoid colon D. Exacerbation of chronic adnexitis E. Follicular cyst of the left ovary 94.Vaginal inspection of a parturient woman revealed: cervix dilation is up to 2 cm, fetal bladder is intact. Sacral cavity is free, sacral promontory is reachable only with a bent finger, the inner surface of the sacrococcygeal joint is accessible for examination. The fetus has cephalic presentation. Sagittal suture occupies the transverse diameter of pelvic inlet, the small fontanel to the left, on the side. What labor stage is this? A. Cervix dilatation stage B. Prodromal stage C. Placental stage D. Stage of fetus expulsion
  • 25. E. Preliminary stage 95.A 68-year-old patient consulted a doctor about a tumour in her left mammary gland. Objectively: in the upper internal quadrant of the left mammary gland there is a neoplasm up to 2,5 cm in diameter, dense, uneven, painless on palpation. Regional lymph nodes are not enlarged. What is the most likely diagnosis? A. Cancer B. Lipoma C. Fibroadenoma D. Cyst E. Mastopathy 96.A 40-year-old female patient has been observing profuse menses accompanied by spasmodic pain in the lower abdomen for a year. Bimanual examination performed during menstruation revealed a dense formation up to 5 cm in diameter in the cervical canal. Uterus is enlarged up to 5-6 weeks of pregnancy, movable, painful, of normal consistency. Appendages are not palpable. Bloody discharges are profuse. What is the most likely diagnosis? A. Nascent submucous fibromatous node B. Cervical myoma C. Algodismenorrhea D. Abortion in progress E. Cervical carcinoma 97. A 29-year-old patient complains of sterility. Sexual life is for 4 years being married, does not use contraception. There was no pregnancy before. On physical examination, genitals are developed normally. Uterine tubes are passable. Rectal temperature during three menstrual cycles is monophase. What is the most probable reason for sterility? A. Anovulatory menstrual cycle B. Genital endometriosis C. Anomalies of genitals development D. Chronic adnexitis E. Immunologic sterility 98.A 45 y.o. woman complains of contact bleedings during 5 months. On speculum examination: hyperemia of uterus cervix, looks like cauliflower, bleeds on probing. On bimanual examination: cervix is of densed consistensy, uterus body isnt enlarged, mobile, nonpalpable adnexa, parametrium is free, deep fornixes. What is the most likely diagnosis? A. Cancer of cervix of uterus B. Cancer of body of uterus
  • 26. C. Cervical pregnancy D. Polypose of cervix of uterus E. Fibromatous node which is being born 99.10 minutes after delivery a woman discharged placenta with a tissue defect 5×6 cm large. Discharges from the genital tracts were profuse and bloody. Uterus tonus was low, fundus of uterus was located below the navel. Examination of genital tracts revealed that the uterine cervix, vaginal walls, perineum were intact. There was uterine bleeding with following blood coagulation. Your actions to stop the bleeding: A. To make manual examination of uterine cavity B. Toadminister uterotonics C. Tointroduce an ether-soaked tampon into the posterior fornix D. Toapply hemostatic forceps upon the uterine cervix E. Toput an ice pack on the lower abdomen 100. On the 5th day after labor body temperature of a 24-year-old parturient suddenly rose up to 38,7°C. She complains about weakness, headache, abdominal pain, irritability. Objectively: AP- 120/70 mm Hg, Ps- 92 bpm, to- 38,7°C. Bimanual examination revealed that the uterus was enlarged up to 12 weeks of pregnancy, it was dense, slightly painful on palpation. Cervical canal lets in 2 transverse fingers, discharges are moderate, turbid, with foul smell. In blood: skeocytosis, lymphopenia, ESR - 30 mm/h. What is the most likely diagnosis? A. Endometritis B. Metrophlebitis C. Lochiometra D. Parametritis E. Pelviperitonitis 101. A 20 y.o. patient complains of amenorrhea. Objectively: hirsutism, obesity with fat tissue prevailing on the face, neck, upper part of body. On the face there are acne vulgaris, on the skin - striae cutis distense. Psychological and intellectual development is normal. Gynecological condition: external genitals are moderately hairy, acute vaginal and uterine hypoplasia. What diagnosis is the most probable? A. Itsenko-Cushing syndrome B. Turners syndrome C. Shichans syndrome D. Babinski-Froehlich syndrome E. Stein-Leventals syndrome
  • 27. 102. A 27 y.o. woman suffers from pyelonephritits of the only kidney. She presents to the maternity welfare centre because of suppresion of menses for 2,5 months. On examination pregnancy 11 weeks of gestation was revealed. In urine: albumine 3,3 g/L, leucocytes cover the field of vision. What is doctors tactics in this case? A. Immediate pregancy interruption B. Pregnancy interruption at 24-25 weeks C. Maintenance of pregnancy till delivery term D. Pregnancy interruption after urine normalization E. Maintenance of pregnancy till 36 weeks 103. An 18-year-old primigravida in her 27-28 week of gestation underwent an operation on account of acute phlegmonous appendicitis. In the postoperative period it is necessary to take measures for prevention of the following pregnancy complication: A. Noncarrying of pregnancy B. Late gestosis C. Fetus hypotrophy D. Intestinal obstruction E. Premature placenta detachment 104. A 24-year-old female patient complains of acute pain in the lower abdomen that turned up after a physical stress. She presents with nausea, vomiting, dry mouth and body temperature 36,6°C. She has a right ovarian cyst in history. Bimanual examination reveals that uterus is dense, painless, of normal size. The left fornix is deep, uterine appendages arent palpable, the right fornix is contracted. There is a painful formation on the right of uterus. Its round, elastic and mobile. It is 7×8 cm large. In blood: leukocytosis with the left shit. What is the most likely diagnosis? A. Ovarian cyst with pedicle torsion B. Acute metritis C. Extrauterine pregnancy D. Right-sided pyosalpinx E. Subserous fibromyoma of uterus 105. A parturient woman is 23 years old. Vaginal obstetric examination reveals full cervical dilatation. There is no fetal bladder. Fetal head is in the plane of pelvic outlet. Sagittal suture is in mesatipellic pelvis, anterior fontanel is closer to pubes. The fetal head diameter in such presentation will be: A. Suboccipito-bregmaticus B. Suboccipitio-frontalis C. Mento-occipitalis
  • 28. D. Fronto-occipitalis recta E. Biparietal 106. A pregnant 26-year-old woman was admitted to a hospital for abdominal pain and bleeding from the genital tract. Bimanual examination revealed that uterus was the size of 9 weeks of pregnancy, the cervical canal let a finger through. Fetal tissues could be palpated in the orifice. There was moderate vaginal bleeding. What is the tactics of choice? A. Instrumental extraction of fetal tissue B. Therapy for the maintenance of pregnancy C. Administration of hormones D. Surveillance E. Hemostatic and antianemic therapy 107. A 42-year-old woman has had hyperpolymenorrhea and progressing algodismenorrhea for the last 10 years. Gynaecological examination revealed no changes of uterine cervix; discharges are moderate, of chocolate colour, uterus is slightly enlarged and painful, appendages are not palpable, the fornices are deep and painless. What is the most likely diagnosis? A. Uterine endometriosis B. Uterine carcinoma C. Endomyometritis D. Adnexal endmetriosis E. Subserous uterine fibromyoma 108. On the tenth day after discharge from the maternity house a 2-year-old patient consulted a doctor about body temperature rise up to 39°C, pain in the right breast. Objectively: the mammary gland is enlarged, there is a hyperemized area in the upper external quadrant, in the same place there is an ill-defined induration, lactostasis, fluctuation is absent. Lymph nodes of the right axillary region are enlarged and painful. What is the most likely diagnosis? A. Lactational mastitis B. Erysipelas C. Abscess D. Dermatitis E. Tumour 109. During the dynamic observation over a parturient woman in the second stage of labor it was registered that the fetal heart rate fell down to 90- 100/min and didnt come to normal after contractions. Vaginal examination revealed the complete cervical dilatation, the fetal head filling the entire posterior surface of the pubic symphysis and sacral hollow; the sagittal suture
  • 29. lied in the anteroposterior diameter of the pelvic outlet, the posterior fontanelle was in front under the pubic arch. What plan for further labour management should be recommended? A. Application of forceps minor B. Episiotomy C. Caesarean section D. Application of cavity forceps E. Stimulation of labour activity by intravenous injection of oxytocin 110. A 27-year-old sexually active female complains of numerous vesicles on the right sex lip, itch and burning. Eruptions regularly turn up before menstruation and disappear 8-10 days later. What is the most likely diagnosis? A. Herpes simplex virus B. Primary syphilis C. Bartholinitis D. Cytomegalovirus infection E. Genital condylomata 111. A 36-year-old female pesented to a gynecological hospital with a significant bleeding from the genital tract and a 1-month delay of menstruation. Bimanual examination revealed soft barrel-shaped cervix. Uterus was of normal size, somewhat softened. Appendages were unremarkable on both sides. Speculum examination revealed that the cervix was cyanotic, enlarged, with the the external orifice disclosed up to 0,5 cm. Urine hCG test was positive. What is the most likely diagnosis? A. Cervical pregnancy B. Uterogestation C. Threatened miscarriage D. Ectopic pregnancy E. Abortion in progress 112. A 26-year-old woman complains of having bloody discharges from the genitals for the last 14 days, abdominal pain, general fatiguability, weakness, weight loss, body temperature rise, chest pain, obstructed respiration. 5 weeks ago she underwent induced abortion in the 6-7 week of gestation. Objectively: the patient is pale and inert. Bimanual examination revealed that the uterus was enlarges up to 8-9 weeks of gestation. In blood: Hb- 72 g/l. Urine test for chorionic gonadotropin gave the positive result. What is the most likely diagnosis? A. Chorioepithelioma B. Uterus perforation C. Uterine carcinoma D. Uterine fibromyoma
  • 30. E. Metroendometritis 113. A 28-year-old patient complains of discomfort, acute pain in the lower third of the left labia majora. The disease began suddenly after menstruation. Objectively: body temperature is 38°C. The left labia majora has a formation to 3 cm diameter, with hyperemic surface, extremely painful to the touch, with symptoms of fluctuation. What is the most likely diagnosis? A. Acute bartholinitis B. Vulvar fibroid C. Vulvar cancer D. Bartholin gland cyst E. Hypertrophy of the labia 114. A 28-years-old woman complains of nausea and vomiting about 10 times per day. She has been found to have body weight loss and xerodermia. The pulse is 100 bpm. Body temperature is 37,2°C. Diuresis is low. USI shows 5-6 weeks of pregnancy. What is the most likely diagnosis? A. Moderate vomiting of pregnancy B. I degree preeclampsia C. Food poisoning D. Premature abortion E. Mild vomiting of pregnancy 115. A 40 week pregnant secundipara is 28 years old. Contractions are very active. Retraction ring is at the level of navel, the uterus is hypertonic, in form of hourglass. On auscultation the fetal heart sounds are dull, heart rate is 100/min. AP of the parturient woman is 130/80 mm Hg. What is the most likely diagnosis? A. Risk of hysterorrhexis B. Attack of eclampsia C. Disturbed labour D. Mazolysis E. Complete hysterorrhexis 116. After delivery and revision of placenta there was found the defect of placental lobule. General condition of woman is normal, uterus is firm, there is moderate bloody discharge. Speculum inspection of birth canal shows absence of lacerations and raptures. What action is nesessary? A. Manual exploration of the uterine cavity B. External massage of uterus C. Urine drainage, cold on the lower abdomen D. Introduction of hemostatic medications E. Introduction of uterine contracting agents
  • 31. 117. A 25 y.o. patient complains of body temperature rise up to 37°C, pain at the bottom of her abdomen and vaginal discharges. Three days ago, when she was in her 11th week of pregnancy, she had an artificial abortion. Objectibely: cervix of uterus is clean, uterus is a little bit enlarged in size, painful. Appendages cannot be determined. Fornixes are deep, painless. Vaginal discharges are sanguinopurulent. What is the most probable diagnosis? A. Postabortion endometritis B. Postabortion uterus perforation C. Parametritis D. Hematometra E. Pelvic peritonitis 118. A 25 y.o. pregnant woman in her 34th week was taken to the maternity house in grave condition. She complains of headache, visual impairment, nausea. Objectively: solid edemata, AP- 170/130 mm Hg. Suddenly there appeared fibrillary tremor of face muscles, tonic and clonic convulsions, breathing came to a stop. After 1,5 minute the breathing recovered, there appeared some bloody spume from her mouth. In urine: protein - 3,5 g/L. What is the most probable diagnosis? A. Eclampsia B. Cerebral hemorrhage C. Epilepsy D. Cerebral edema E. Stomach ulcer 119. A 51-year-old patient complains of having intensive bloody discharges from vagina for 15 days after delay of menstruation for 2,5 months. In anamnesis: disorders of menstrual function during a year, at the same time she felt extreme irritability and had sleep disorders. US examination results: uterus corresponds with age norms, appendages have no pecularities, endometrium is 14 mm thick. What is the doctors tactics? A. Diagnostic curettage of uterine cavity B. TORCH-infection test C. Hysterectomy D. Conservative treatment of bleeding E. Supravaginal amputation of uterus without appendages 120. An 18 y.o. patient complains of painfulness and swelling of mammary glands, headaches, irritability, edemata of lower extremities. These symptoms have been present since the begin of menarche, appear 3-4 days before regular menstruation. Gynecological examination revealed no pathology. What is the most probable diagnosis?
  • 32. A. Premenstrual syndrome B. Neurasthenia C. Mastopathy D. Disease of cardiovascular system E. Renal disease 121. A 22-year-old female patient complains of dull pain in her right iliac area that she has been experiencing for a week, morning sickness and gustatory change. She has a history of menstruation delay for 3 weeks. Objectively: AP- 80/50 mm Hg, pulse is 78 bpm, body temperature is 37°C. Bimanual examination reveals that uterus is enlarged, soft, mobile and painless. Uterine appendages are palpable on the right, there is a dense, elastic and moderately painful formation 3x4 cm large. What is the most likely diagnosis? A. Progressing fallopian pregnancy B. Right ovarian cyst C. Acute appendicitis D. Uterogestation E. Interrupted fallopian pregnancy 122. A 30 y.o. parturient woman was taken to the maternity house with complaints of having acute, regular labour pains that last 25-30 seconds every 1,5-2 minutes. Labour activity began 6 hours ago. Uterus is in higher tonus, head of the fetus is above the opening into the small pelvis. Fetal heartbeat is 136/min. P.V: cervical dilatation is 4 cm, uterine fauces is spasming at a height of parodynia. Head is level with opening into the small pelvis, it is being pushed off. What is the most probable diagnosis? A. Discoordinated labour activity B. Pathological preliminary period C. Normal labour activity D. Primary powerless labour activity E. Secondary powerless labour activity 123. A primigravida woman appealed to the antenatal clinic on the 22.03.03 with complaints of boring pain in the lower part of abdomen. Anamnesis registered that her last menstruation was on the 4.01.03. Bimanual examination revealed that uterine servix is intact, external fauces is closed, uterus is enlarged up to the 9-th week of pregnancy, movable, painless. What complication can be suspected? A. Risk of abortion in the 9-th week of pregnancy B. Hysteromyoma C. Vesicular mole D. Abortion that started in the 9-th week of pregnancy
  • 33. 124. A 25-year-old female patient complains about having amenorrhea for 3 years. She associates it with difficult labour complicated by massive hemorrhage. She also complains of loss of weight, hair fragility and loss, lack of appetite and depression. Objective examination reveals no pathological changes of uterus and its appendages. What is the disease pathogenesis? A. Hypoproduction of gonadotropin B. Hyperproduction of estrogens C. Hypoproduction of progesterone D. Hyperproduction of prolactin E. Hyperproduction of androgens 125. A 30-year-old patient consulted a doctor about menstruation absence for 2 years after labour, loss of hair, body weight loss. The labour was complicated by a haemorrhage caused by uterus hypotonia. Objectively: the patient is asthenic, external genitals are hypoplastic, the uterus body is small and painless. The appendages are not palpaple. What is the most likely diagnosis? A. Sheehans syndrome B. Turners syndrome C. Galactorrhea-amenorrhea syndrome D. Exhausted overy syndrome E. Ovarian amenorrhea 126. A 28-year-old patient has been admitted to the gynecological department three days after a casual coitus. She complains about pain in her lower abdomen and during urination, profuse purulent discharges from the vagina, body temperature rise up to 37,8°C. The patient was diagnosed with acute bilateral adnexitis. Supplemental examination revealed: the 4th degree of purity of the vaginal secretion, leukocytes within the whole visual field, diplococcal bacteria located both intra- and extracellularly. What is the etiology of acute adnexitis in this patient? A. Gonorrheal B. Chlamydial C. Staphylococcal D. Trichomonadal E. Colibacterial 127. A 25-year-old woman came to a maternity welfare clinic and complained about being unable to conceive within 3 years of regular sexual life. Examination revealed weight gain, male pattern of hair distribution on the pubis, excessive pilosis of thighs. Ovaries were dense and enlarged, basal temperature was monophase. What is the most likely diagnosis?
  • 34. A. Sclerocystosis of ovaries B. Adrenogenital syndrome C. Gonadal dysgenesis D. Premenstrual syndrome E. Tubo-ovaritis 128. A 28 year old patient complained about prolongation of intermenstrual periods up to 2 months, hirsutism. Gynaecological examination revealed that the ovaries were enlarged, painless, compact, uterus had no pecularities. Pelvic ultrasound revealed that the ovaries were 4-5 cm in diameter and had multiple enlarged follicles on periphery. Roentgenography of skull base showed that sellar region was dilated. What is the most probable diagnosis? A. Stein-Leventhal syndrome B. Sheehans syndrome C. Morgagni-Stewart syndrome D. Premenstrual syndrome E. Algodismenorrhea 129. A 32-year-old gravida complains of episodes of unconsciousness, spontaneous syncopes that are quickly over after a change of body position. A syncope can be accompanied by quickly elapsing bradycardia. There are no other complications of gestation. What is the most likely reason for such condition? A. Postcava compresseion by the gravid uterus B. Vegetative-vascular dystonia (cardiac type) C. Psychosomatic disorders D. Pressure rise in the veins of extremities E. Pressure fall in the veins of extremities 130. A woman consulted a therapeutist about fatigability, significant weight loss, weakness, loss of appetite. She has had amenorrhea for 8 months. A year ago she born a full-term child. Haemorrhage during labour made up 2 l. She got blood and blood substitute transfusions. What is the most probable diagnosis? A. Sheehans syndrome B. Shereshevsky-Turners syndrome C. Vegetovascular dystonia D. Homological blood syndrome E. Stein-Leventhal syndrome 131. A 54-year-old female patient consulted a gynaecologist about bloody discharges from the vagina for 1 month. Last menstruation was 5 years ago.
  • 35. Gynaecological examination revealed no pathological changes. What is the tactics of choice? A. Diagnostic fractional curettage of uterine cavity B. USI C. Colposcopy D. Cytosmear E. Symptomatic therapy 132. A 28-year-old female patient complains of having haemorrhage from the genital tracts for 1 month. 6 months ago she had natural delivery and gave birth to a girl weighing 3100 g. Objectively: the uterus is enlarged to 9-10 weeks, mobile, painless, of heterogenous consistency. Examination reveals vaginal cyanosis, anaemia and body temperature rise up to 37,8°C. There is a significant increase in hCG concentration in the urine. What is your provisional diagnosis? A. Uterine chorionepithelioma B. Endometritis C. Uterine fibromyoma D. Pregnancy E. Hydatidiform mole 133. An ambulance delivered a 21-year-old woman to the gynaecological department with complaints of colicky abdominal pain and bloody discharges from the genital tracts. Bimanual examination revealed that uterus was soft, enlarged to the size of 6 weeks of gestation, a gestational sac was palpated in the cervical canal. Uterine appendages werent palpable. Fornices are free, deep and painless. Discharges from the genital tracts are bloody and profuse. What is the most likely diagnosis? A. Abortion in progress B. Interrupted fallopian pregnancy C. Threat of abortion D. Cervical pregnancy E. Incipient abortion 134. A 33 y.o. woman survived twooperations on account of extrauterine pregnancy, both uterine tubes were removed. She consulted a doctor with a question about possibility of having a child. What can be advised in this case? A. Extracorporal fertilization B. Induction of ovulation C. Substitutional maternity D. Insemination with her husbands semen E. Artifical fertilization with donors semen
  • 36. 135. On the fifth day after a casual sexual contact a 25-year-old female patient consulted a doctor about purulent discharges from the genital tracts and itch. Vaginal examination showed that vaginal part of uterine cervix was hyperemic and edematic. There was an erosive area around the external orifice of uterus. There were mucopurulent profuse discharges from the cervical canal, uterine body and appendages exhibited no changes. Bacterioscopic examination revealed bean-shaped diplococci that became red after Grams staining. What is the most likely diagnosis? A. Acute gonorrheal endocervicitis B. Candidal vulvovaginitis C. Bacterial vaginism D. Clamydial endocervicitis E. Trichomonal colpitis 136. A 26 year old woman complains about edemata, swelling and painfulness of mammary glands, headache, tearfulness, irritability. These signs turn up 5 days before menstruation and disappear after its start. What clinical syndrome is it? A. Premenstrual syndrome B. Stein-Leventhal syndrome C. Adrenogenital syndrome D. Postcastration syndrome E. Climacteric syndrome 137. A 49-year-old woman complains about headache, head and neck going hot, increased perspiration, palpitation, arterial pressure rise up to 170/100 mm Hg, irritability, insomnia, tearfulness, memory impairment, rare and scarce menses, body weight increase by 5 kg over the last half a year. What is the most likely diagnosis? A. Climacteric syndrome B. Arterial hypertension C. Postcastration syndrome D. Premenstrual syndrome E. Vegetative-vascular dystonia 138. A 30-year-old female patient has been delivered to the gynaecological department with complaints of acute pain in the lower abdomen and body temperature 38,8°C. In history: sexual life out of wedlock and two artificial abortions. Gynaecological examination reveals no changes of uterine. The appendages are enlarged and painful on both sides. Vaginal discharges are purulent and profuse. What study is required to confirm a diagnosis? A. Bacteriological and bacterioscopic analysis B. Colposcopy C. Laparoscopy
  • 37. D. Hysteroscopy E. Curettage of uterine cavity 139. A parturient woman is 27 year old, it was her second labour, delivery was at term, normal course. On the 3rd day of postpartum period body temperature is 36,8°C, Ps - 72/min, AP - 120/80 mm Hg. Mammary glands are moderately swollen, nipples are clean. Abdomen is soft and painless. Fundus of uterus is 3 fingers below the umbilicus. Lochia are bloody, moderate. What is the most probable diagnosis? A. Physiological course of postpartum period B. Lactostasis C. Postpartum metroendometritis D. Subinvolution of uterus E. Remnants of placental tissue after labour 140. A parturient woman is 25 years old, it is her second day of postpartum period. It was her first full-term uncomplicated labour. The lochia should be: A. Bloody B. Mucous C. Sanguino-serous D. Purulent E. Serous 141. A 32-year-old patient consulted a doctor about being inable to get pregnant for 5-6 years. 5 ago the primipregnancy ended in artificial abortion. After the vaginal examination and USI the patient was diagnosed with endometrioid cyst of the right ovary. What is the optimal treatment method? A. Surgical laparoscopy B. Conservative therapy with estrogen-gestagenic drugs C. Sanatorium-and-spa treatment D. Hormonal therapy with androgenic hormones E. Anti-inflammatory therapy 142. A woman is 34 years old, it is her tenth labor at full term. It is known from the anamnesis that the labor started 11 hours ago, labor was active, painful contractions started after discharge of waters and became continuous. Suddenly the parturient got knife-like pain in the lower abdomen and labor activity stopped. Examination revealed positive symptoms of peritoneum irritation, ill-defined uterus outlines. Fetus was easily palpable, movable. Fetal heartbeats wasnt auscultable. What is the most probable diagnosis? A. Rupture of uterus B. II labor period C. Discoordinated labor activity
  • 38. D. Uterine inertia E. Risk of uterus rupture 143. Examination of placenta revealed a defect. An obstetrician performed manual investigation of uterine cavity, uterine massage. Prophylaxis of endometritis in the postpartum period should involve following actions: A. Antibacterial therapy B. Contracting agents C. Intrauterine instillation of dioxine D. Instrumental revision of uterine cavity E. Haemostatic therapy 144. A 10 week pregnant woman was admitted to a hospital for recurrent pain in the lower abdomen, bloody discharges from the genital tracts. The problems turned up after ARVI. The woman was registered for antenatal care. Speculum examination revealed cyanosis of vaginal mucosa, clean cervix, open cervical canal discharging blood and blood clots; the lower pole of the gestational sac was visible. What tactics should be chosen? A. Curettage of the uterus B. Expectant management, surveillance C. Pregnancy maintenance therapy D. Hysterectomy E. Antiviral therapy 145. A 30 year old patient complains about inability to become pregnant over 3 years of married life. The patient is of supernutrition type, she has hair along the median abdominal line, on the internal thigh surface and in the peripapillary area. Menses started at the age of 16, they are infrequent and non-profuse. US revealed that the uterus was of normal size, ovaries were 4?5?5 cm large and had a lot of cystic inclusions. What is the most probable diagnosis? A. Polycystic ovaries B. Ovarian cystoma C. Menstrual irregularity D. Bilateral ovarian tumours E. Chronic oophoritis 146. A 29-year-old patient complains of absent menstruation for a year, milk discharge from the nipples when pressed, loss of lateral visual fields. X- ray shows an expansion of the sella turcica. What is the most likely cause of this condition? A. Pituitary tumour B. Pregnancy
  • 39. C. Functional disorder of the hypothalamic-pituitary-ovarian system D. Mammary tumour E. Ovarian tumor 147. A patient with fibromyoma of uterus sized up to 8-9 weeks of pregnancy consulted a gynaecologist about acute pain in the lower abdomen. Examination revealed pronounced positive symptoms of peritoneal irritation, high leukocytosis. Vaginal examination revealed that the uterus was enlarged corresponding to 9 weeks of pregnancy due to the fibromatous nodes, one of which was mobile and extremely painful. Appendages were not palpable. There were moderate mucous discharges. What is the optimal treatment tactics? A. Urgent surgery (laparotomy) B. Fractional diagnostic curettage of the uterine cavity C. Surveillance and spasmolytic therapy D. Surgical laparoscopy E. Surveillance and antibacterial therapy 148. A pregnant woman was delivered to the gynecological unit with complaints of pain in the lower abdomen and insignificant bloody discharges from the genital tracts for 3 hours. Last menstruation was 3 months ago. Vaginal examination showed that body of womb was in the 10th week of gestation, a fingertip could be inserted into the external orifice of uterus, bloody discharges were insignificant. USI showed small vesicles in the uterine cavity. What is the most likely diagnosis? A. Grape mole B. Abortion in progress C. Threat of spontaneous abortion D. Incomplete abortion E. Incipient abortion 149. A 49-year-old patient undergoes regular medical check-up for uterine fibromyoma. Within the last year the uterus has enlarged up to 20 weeks of gestation. What is the rational way of treatment? A. Surgical treatment B. Treatment with prostaglandin inhibitors C. Further surveillance D. Hormonal therapy E. Embolization of uterine arteries 150. A female patient complains of being unable to get pregnant for 5 years. A complete clinical examination brought the following results: hormonal function is not impaired, urogenital infection hasnt been found, on
  • 40. hysterosalpingography both tubes were filled with the contrast medium up to the isthmic segment, abdominal contrast was not visualized. The patients husband is healthy. What tactics will be most effective? A. In-vitro fertilization B. Laparoscopic tubal plasty C. ICSI within in-vitrofertilization program D. Insemination with husbands sperm E. Hydrotubation 151. A 19-year-old primiparous woman with a body weight of 54,5 kg gave birth at 38 weeks gestation to a full-term live girl after a normal vaginal delivery. The girls weight was 2180,0 g, body length - 48 cm. It is known from history that the woman has been a smoker for 8 years, and kept smoking during pregnancy. Pregnancy was complicated by moderate vomiting of pregnancy from 9 to 12 weeks pregnant, edemata of pregnancy from 32 to 38 weeks. What is the most likely cause of low birth weight? A. Fetoplacental insufficiency B. Third trimester preeclampsia C. Womans age D. Low weight of the woman E. First trimester preeclampsia 152. A primigravida is 22 years old. She has Rh(-), her husband has Rh(+). Antibodies to Rh werent found at 32 weeks of pregnancy. Redetermination of antibodies to Rh didnt reveal them at 35 weeks of pregnancy as well. How often should the antibodies be determined hereafter? A. Once a week B. Once in three weeks C. Once in two weeks D. Montly E. There is no need in further checks 153. A maternity house has admitted a primagravida complaining of irregular, intense labour pains that have been lasting for 36 hours. The woman is tired, failed to fall asleep at night. The fetus is in longitudinal lie, with cephalic presentation. The fetus heartbeat is clear and rhythmic, 145/min. Vaginal examination revealed that the uterine cervix was up to 3 cm long, dense, with retroflexion; the external orifice was closed; the discharges were of mucous nature. What is the most likely diagnosis? A. Pathological preliminary period B. Physiological preliminary period C. Secondary uterine inertia D. Uterine cervix dystocia E. Primary uterine inertia
  • 41. 154. A 14-year-old girl complains of pain in vaginal area and lower abdomen that last for 3-4 days and have been observed for 3 months about the same time. Each time pain is getting worse. Objectively: mammary glands are developed, hairiness corresponds to the age. The virginal membrane is intact, cyanotic and protruded. She has never had menstruation. She has been diagnosed with primary amenorrhea. What is the reason of amenorrhea? A. Hymen atresia B. Sexual development delay C. Babinski-Frohlich syndrome D. Turners syndrome E. Pregnancy 155. A multigravida with Rh-isosensitization was found to have a decrease in anti-Rh titer from 1:32 to 1:8 at 33-34 weeks of gestation. Ultrasound revealed double contour of head, ebnlargement of fetal liver, placental thickness of 50 mm. The patient has indication for: A. Premature delivery B. Plasmapheresis C. Administration of anti-Rh gamma globulin D. Repeated (after 2 weeks) USI E. Course of desensitizing therapy 156. A 13-year-old girl was admitted to the gynecological department with heavy bleeding, which appeared after a long delay of menstruation. Shortly before, the girl suffered a serious psychotrauma. Her menarche occurred at the age of 11, she has a 30-day cycle with 5 to 6 days of moderate, painless bleeding. The patient is somatically healthy, of normosthenic constitution with height of 160 cm, weight of 42 kg. The patient is pale. Rectoabdominal examination revealed that the uterus was of normal size and consistency, anteflexio-versio, the appendages were not changed. What is the most likely diagnosis? A. Juvenile bleeding B. Hysteromyoma C. Amenorrhea D. Girl is healthy E. Ovarian cyst 157. A 38-year-old female patient complains about hot flashes and feeling of intense heat arising up to 5 times a day, headaches in the occipital region along with high blood pressure, palpitations, dizziness, fatigue, irritability, memory impairment. 6 months ago the patient underwent extirpation of the uterus with its appendages. What is the most likely diagnosis?
  • 42. A. Post-castration syndrome B. Premenstrual syndrome C. Secondary psychogenic amenorrhea D. Physiological premenopause E. Early pathological menopause 158. A 27-year-old patient complains of irritability, tearfulness, depression, and sometimes aggressiveness, headache, nausea, vomiting, swelling of the mammary glands. The mentioned problems arise 5-6 days before menstruation and gradually progress until menstruation, 3 days after it the problems disappear. What is the most likely diagnosis? A. Premenstrual syndrome B. Secondary psychogenic amenorrhea C. Premature pathological climacterium D. Preclimacterium syndrome E. Algomenorrhea 159. A 23-year-old primigravida at 39 weeks gestation has been admitted to the maternity ward with irregular contractions. The intensity of uterine contractions is not changing, the intervals between them stay long. Bimanual examination reveals that the cervix is centered, soft, up to 1,5 cm long. There is no cervical dilatation. What diagnosis should be made? A. Pregnancy I, 39 weeks, preliminary period B. Pregnancy I, 39 weeks, labor I, period 1, the active phase C. Pregnancy I, 39 weeks, pathological preliminary period D. Pregnancy I, 39 weeks, birth I, 1 period, the acceleration phase E. Pregnancy I, 39 weeks, labor I, 1 period, the latent phase 160. 20 minutes after a normal delivery at 39 weeks a puerpera had a single temperature rise up to 38°C. Objectively: the uterus is dense, located between the navel and the pubis, painless. Lochia are bloody, of small amount. Breasts are moderately soft and painless. What is the optimal tactics? A. Further follow-up B. Appointment antipyretic C. Expression of breast D. Manual examination of the uterine cavity E. Antibiotic therapy 161. On the 10th day postpartum a puerperant woman complains of pain and heaviness in the left breast. Body temperature is 38,8°C, Ps - 94 bpm. The left breast is edematic, the supero-external quadrant of skin is hyperemic. Fluctuation symptom is absent. The nipples discharge drops of milk when pressed. What is a doctors further tactics?
  • 43. A. Antibiotic therapy, immobilization and expression of breast milk B. Physiotherapy C. Opening of the abscess and drainage of the breast D. Compress to both breasts E. Inhibition of lactation 162. On the 10th day postpartum a puerperant woman complains of pain and heaviness in the left mammary gland. Body temperature is 38,8°C, Ps- 94 bpm. The left mammary gland is edematic, the supero-external quadrant of skin is hyperemic. Fluctuation symptom is absent. The nipples discharge drops of milk when pressed. What is a doctors further tactics? A. Antibiotic therapy, immobilization and expression of breast milk B. Physiotherapy C. Opening of the abscess and drainage of the mammary gland D. Compress to both mammary glands E. Inhibition of lactation 163. A 30-year-old female patient complains of milk discharge from the mammary glands, 5-month absence of menstruation. She had one physiological labour four years ago. Objectively: mammary glands are normally developed. Bimanual examination reveals that the uterus is decreased in size, the ovaries are of normal size. MRI-scan shows no cerebral pathologies. Concentration of thyroid-stimulating hormone is normal. The serum prolactin level is increased. What is the most likely diagnosis? A. Hyperprolactinemia B. Hypothyroidism C. Pituitary adenoma D. Sheehan syndrome E. Polycystic ovary syndrome 164. During self-examination a 22-year-old patient revealed a mammary tumour. Palpation revealed a firm, painless, mobile formation up to 2 cm, peripheral lymph nodes were not changed. USI results: in the superior external quadrant of the right mammary gland there was a big formation of increased echogenicity, sized 18x17 mm. The patient was provisionally diagnosed with fibroadenoma. What is a doctors further tactics? A. Surgical removal of the tumour prior to pregnancy B. Dynamic follow-up C. Radical mastectomy D. Nonsteroid anti-inflammatory drugs, oral contraceptives E. Surgical treatment after pregnancy
  • 44. 165. A 25-year-old female has a self-detected tumor in the upper outer quadrant of her right breast. On palpation there is a painless, firm, mobile lump up to 2 cm in diameter, peripheral lymph nodes are not changed. In the upper outer quadrant of the right breast ultrasound revealed a massive neoplasm with increased echogenicity sized 21x18 mm. What is the most likely diagnosis? A. Fibroadenoma B. Diffuse mastopathy C. Mastitis D. Mammary cancer E. Lactocele 166. A 49-year-old female patient complains of itching, burning in the external genitals, frequent urination. The symptoms have been present for the last 7 months. The patient has irregular menstruation, once every 3-4 months. Over the last two years she has had hot flashes, sweating, sleep disturbance. Examination revealed no pathological changes of the internal reproductive organs. Complete blood count and urinalysis showed no pathological changes. Vaginal smear contained 20-25 leukocytes per HPF, mixed flora. What is the most likely diagnosis? A. Menopausal syndrome B. Trichomonas colpitis C. Cystitis D. Vulvitis E. Bacterial vaginosis 167. 2 weeks after labour a parturient woman developed breast pain being observed for 3 days. Examination revealed body temperature at the rate of 39°C, chills, weakness, hyperaemia, enlargement, pain and deformity of the mammary gland. On palpation the infiltrate was found to have an area of softening and fluctuation. What is the most likely diagnosis? A. Infiltrative-purulent mastitis B. Phlegmonous mastitis C. Serous mastitis D. Mastopathy E. Lactostasis 168. Preventive examination of a 50-year-old woman revealed a dense tumour of the right mammary gland up to 5 cm in diameter without distinct outlines. The skin over the tumour looked like lemon peel. Palpation revealed a lymph node in the axillary region. What is the most likely diagnosis? A. Breast cancer B. Mastitis C. Breast lipoma
  • 45. D. Lactocele E. Diffuse mastopathy 169. A 20-year-old female consulted a gynecologist about not having menstrual period for 7 months. History abstracts: early childhood infections and frequent tonsillitis, menarche since 13 years, regular monthly menstrual cycle of 28 days, painless menstruation lasts 5-6 days. 7 months ago the patient had an emotional stress. Gynecological examination revealed no alterations in the uterus. What is the most likely diagnosis? A. Secondary amenorrhea B. Spanomenorrhea C. Cryptomenorrhea D. Primary amenorrhea E. Algomenorrhea 170. A 48-year-old female has been admitted to the gynecology department for pain in the lower right abdomen and low back pain, constipations. Bimanual examination findings: the uterus is immobile, the size of a 10-week pregnancy, has uneven surface. Aspirate from the uterine cavity contains atypical cells. What diagnosis can be made? A. Hysterocarcinoma B. Colon cancer C. Chorionepithelioma D. Cervical cancer E. Metrofibroma